Sunteți pe pagina 1din 136

DR.

FAYZA EXAM
1. The most common side effect of tacrolimus over cyclosporin is
a. More neurotoxic
b. More nephrotoxic
c. More hepatotoxic
d. Gingival hyperplasia

2. Patient with SLE on treatment. She wants to take ant SLE treatment but also wants to
preserve her fertility to get pregnant. She should use prednisolone plus
a. Azathioprine
b. Mycophenolate
c. Cyclosporin
d. Tacrolimus

3. The procedure of veno-venal CRT is suitable for patient with


a. DM
b. ESRD
c. Hypertensive emergency & AKI
d. Malignancy & CKD
e. _________________

4. The most appropriate chemistry (ABGs) for salicylate poisoning is


a. Respiratory alkalosis & metabolic acidosis
b. ______________
c. ______________
d. ______________
5. The most appropriate method to diagnose unilateral renal vein thrombosis is
a. Differential renin measurement in renal artery
b. Differential renin measurement in renal vein
c. Ultrasonography
d. MR/CT venography

6. A 36 years old male came in ER after suicide attempt & ingestion of anti-freeze sloution.
His HCO3 is 12, PCO2= 22, Na=138, K=4.8, Cl=98. The immediate management will be
a. Immediate HD
b. No traetment as there is no more acidosis
c. Aggressive diuresis with diuretics
d. Administration of fomepizole

7. In case of hypernatremia Na=170 & plasma osmolality 320, the most probable condition
will be
a. Increased intracellular volume increased osmolarity >328
b. Decreased ICV and decreased osmolarity < 328
c. Decreased ICV and increased osmolarity 328
d. No change in ICV and osmolarity 420

8. Male refused from military services because of microscopic hematuria, but his sensori-
neural examination & eye examination are normal & his other labs are normal, His brother
has the same symptoms without proteinuria. What is the prognosis in the future
a. Patient will develop CRF in 20 years
b. Needs transfusion at sometime
c. No abnormality will occur usually
d. Will need immediate management
9. Patient with ethanol ingestion fell down & has head trauma. 3 days later he develops
confusion & coma. Electrolytes show hypernatremia (Na=170) from 140 at the time of
admission. Plasma osmolarity is 300. The most common cause
a. Decreased water intake
b. Increased diuresis
c. Nephrogenic DI
d. Central DI
e. Inappropriate saline infusion

10. Patient with microscopic hematuria & proteinuria within few days following upper
respiratory tract infection. Renal biopsy showing
a. Electron dense deposits on electron microscopy
b. IgA deposition in the mesangium by immunoflourecsnce
c. Proliferation& increased matrix of the mesangium with dense deposits
d. All of the above

11. The commonest cause of Normal sized kidney despite ESKD is


a. Scleroderma
b. Polycystic kidney
c. Amyloidosis
d. Chronic analgesic & scarring kidneys

12. The most frequent action of ACEi with unilateral renal artery stenosis is
a. Frequently causes renal failure
b. Is appropriate treatment for hypertension
c. Inhibition of renin activity of the kidney
d. __________________
13. Diabetic patient with peritoneal dialysis develops severe peritonitis and C/S shows
multiple Gram +ve & Gram –ve organisms. The most appropriate investigation needed is
a. Transperitoneal angiography
b. CT
c. USG for the lumen of the catheter
d. X-Ray abdomen

14. Diabetic patient has UTI on treatment. Culture sensitivity, electrolytes & blood chemistry
are normal but computerized tomography shows gas in renal parenchyma. Which treatment to
add
a. Clindamycin
b. Amphotericin B
c. Other antibiotics
d. Emergency nephrectomy

15. Pregnant female with UTI. The most correct Answer is


a. 50% have asymptomatic bacteruria
b. No treatment indicated
c. Urgent IVU to detect cause
d. Observe the patient untill the symptoms appear and then treatment with antibiotics.
e. Immediate propmpt treatment if asymptomatic bacteruria +

16. The best method (gold standard) for estimation of GFR is


a. Cockroft Gault formula
b. 24h creatinine clearance
c. Inulin clearance
d. Modification of diet in renal disease study equation
17. Diabetic patient with ESRD needs to decrease BP less than
a. 130/80 mmHg
b. 139/85 mmHg
c. 140/80 mmHg
d. 150/90 mmHg

18. 5 years boy with ESRD will require HD or Continuous renal replacement therapy when
GFR less than
a. 15%
b. 20%
c. 30%
d. 80%

19. Infant having UTI. The best sensitive investigation for infection
a. High WBC in urine
b. Leukocytosis in blood (CBC)
c. Urea esterase in urine dipstick
d. Presence of leukocytic cast in urine

20. Drugs that decrease cyclosporine levels in the blood is


a. Fluconazole
b. Trimethoprim
c. ___________
d. ___________

21. 65 years old male with microscopic hematuria needs to donate his 54 years old wife with
ESRD. His chemsitry is normal. He asks about your opinion
a. Encourage him as microscopic hematuria is not significant
b. Discourage him for his isolated hematuria
c. ________________
d. ________________

22. The most likely to cause nephrocalcinosis & papillary calcification. All are true except
a. Sarcoidosis
b. Primary hyperparathyroidism
c. NSAIDs
d. Type-I RTA
e. Medullary cystic disease

23. Hemoperfusion is most appropriate treatment in


a. Ethanol intoxication
b. Methanol intoxication
c. Theophylline intoxication
d. NSAIDs intoxication

24. In case of hyponatremia Na=128, the least likely to occur


a. Total body Na decreased
b. Total body Na increased
c. Total body water increased
d. Total body Na & water decreased
e. None of the above

25. Patient with aortic aneurysm, underwent surgery for aneurysm correction, shows
deteriorated kidney function. The most comon cause is
a. Acute tubular necrosis
b. Analgesic nephropathy
c. Cholesterol embolization
d. ___________________
26. CHF on diuretics developed weakness & carpopedal spasm, prolonged QT interval &
depressed ST segment. The most common cause is
a. Hyperkalemia
b. Hypomagnesemia
c. Hyperphosphatemia
d. Hypophosphatemia

27. In rhabdomyolysis all of the following occurs except


a. Hyperkalemia
b. Hyperphosphatemia
c. Hypercalcemia
d. Hyperuricemia
e. Hypocalcemia

28. Drugs that decrease the cyclosporin level


a. Trimethoprim
b. Fluconazole
c. Erythromycin
d. Rifampicin

29. Patient with ESRD due to DM-2 has creatinine clearance <17ml/min/kg. his
transplantation workup shows no obstacles for transplantation. What will be your decision
a. Transplant now
b. Peritoneal dialysis
c. Hemodialysis
d. Transplant of trial of dialysis
30. Renal agenesis is associated with all except
a. Meckel’s diverticulum
b. Imperforate anus
c. Vertebral malformations
d. Cardiac abnormalities

31. All of the following antibiotics are safe in pregnancy except


a. Amoxicillin
b. Quinolone
c. Nitrofurantoin
d. Tetracycline

32. How to diagnose UTI in pregnancy


a. 105 bacteria/ml
b. 1000 to 10000 bacteria/ml
c. ……………………..
d. ……………………..

33. Patient having diarrhea & dehydration & taking antibiotics. How to differentiate the pre-
renal azotemia from antibiotics toxicity
a. ……………………..
b. ……………………..
c. ……………………..
d. ……………………..
34. How to distinguish action of NSAIDs from Antibiotics on kidney
a. Rash & eosinophilia
b. ……………………..
c. ……………………..
d. ……………………..
35. Which is more characteristic of CKD
a. Hypophosphatemia
b. Hypokalemia
c. Hypocalcemia
d. Hypoglycemia

36. What is the acid base disturbance caused by aspirin


a. Respiratory alkalosis & metabolic acidosis

37. What is the mechanism of action of Lithium toxicity on kidney


a. Chronic interstitial nephritis
b. Acute interstitial nephritis
c. Acute urinary obstruction
d. Diabetes mellitus

38. Which is characteristic of ATN


a. Hyaline cast
b. RBCs casts
c. WBC cast
d. Muddy brown cast

39. What is the mechanism of action of trimethoprim Nephrotoxicity


a. ATN
b. Acute interstitial nephritis
c. GN
d. ……………………..
40. Cardiovascular calcification in CKD is due to
a. Hyperparathyroidism/hyperphosphatemia??
b. Hypocalcemia
c. Hypercalcemia
d. High calcium intake
e. High Vitamin D intake

41. A patient developed dusky leg after coronary artery manipulation. What is the most likely
cause
a. Cholesterol embolization

42. Serum creatinine is not exact indicator of GFR, because of all of the following except
a. It varies with weight
b. It varies with diet
c. It varies with age
d. It varies with medication
e. None of the above

43. Which of the following can best lower serum K immediately


a. Ca gluconate
b. Insulin glucose infusion
c. Calcium resonium
d. NaHCO3
e. β2 agonists
44. Pyelonephritis with E. Coli isolated, patient not improving with antibiotics. Radiography
shows gas in renal parenchyma
a. Add clindamycin
b. Add amphotericin B
c. Emergency nephrectomy
d. ………………………….

45. What is the best treatment for DI


a. Desmopressin
b. ……………………..
c. ……………………..
d. ……………………..

46. Patient having renal impairment, hemoptysis & B/L opacities in CXR. What is the most
likely diagnosis
a. Wegener’s granulomatosis
b. Good pasture syndrome
c. ……………………..
d. ……………………..

47. Microscopic hematuria in family (brother & his sister). What is the most likely diagnosis
a. Autosomal recessive Alport’s Syndrome
b. Thin basement membrane disease
c. IgA nephropathy
48. Which of the following is related to chromosome 3
a. Tuberous sclerosis complex
b. ADPKD
c. Von Hippel Lindau disease
d. Recklinghausen disease

49. Diabetic patient on HD with multi access failure, currently having no access. What to do
a. Permanent catheter a or b
b. Venous graft
c. Renal transplantation
d. CAPD
e. Conservative

50. Flail chest, HCO3=15, PCO2=30, pH=7.56


a. Metabolic acidosis
b. Metabolic alkalosis
c. Metabolic acidosis with respiratory alkalosis
d. Metabolic acidosis with respiratory acidosis
e. Respiratory alkalosis partially compensated

51. How to follow up SLE


a. ANA
b. Anti DNA
c. CRP
d. Complement levels
e. Anti Ds DNA +
52. Study HEMO: high flux dialyzer with 1.7=Kt/V and standard dialyzer with Kt/V 1.3.
what can do higher dialysis mortality
a. Increase
b. Decrease
c. No change
d. ……………………..

53. HCO3=30, K=3, urinary potassium=10, pH=7.26, chloride=96. These results are
compatible with
a. GI loss
b. Diuretic abuse
c. Bartter’s syndrome
d. RTA

54. CMV should be diagnosed best with


a. Anti CMV IgG
b. Anti CMV IgM
c. CMV antigen +
d. Isolation of CMV

55. PD: when


a. ……………………..
b. ……………………..
c. Rapid transporter
d. Low transporter
56. In PD, which thing can be better for patients
a. Low protein diet
b. High sugar diet
c. Overload
d. All of the above
e. None of the above

57. Regarding urinary tract TB, all are true except


a. Urinary obstruction
b. Isolation of mycobacteria in urine
c. Painless hematuria +
d. Sterile pyuria

58. In peritoneal dialysis, when you change solute concentration from 2.25% to 4.5%
glucose, the result is
a. ……………………..
b. ……………………..
c. ……………………..
d. ……………………..
e. More UF

59. 60 years old diabetic patient having GFR=17 ml/min/1.73m2. His relative wants to donate
him kidney and has no contraindication. What should be done
a. Transplant now
b. Wait after a trial of dialysis
c. ……………………..
d. ……………………..
e. ……………………..
60. A 63 years old male with microscopic hematuria, Proteinuria less than 100mg/24h. Her
58 years old wife diagnosed ESRD. He asks about your opinion regarding kidney donation
a. Encourage him as hematuria is microscopic
b. Encourage him as hematuria is not associated with Proteinuria
c. Encourage him as he is not hematuria is not associated with HTN
d. Encourage him as hematuria is not associated with renal injury
e. Discourage him as he has isolated hematuria

61. CKD-5 is labeled when GFR is decreased to


a. 50%
b. 35%
c. 25%
d. 15%
e. 5%

62. Patient admitted after myocardial infarction & treated with coronary stenting. Few days
later, he developed a skin rash, oliguria & renal impairment. What is the most likely
diagnosis
a. ATN
b. Acute interstitial nephritis
c. Ac. GN
d. Cholesterol embolization

63. What is the initial cause of renal osteodystrophy in CKD


a. Hypocalcemia
b. Hyperparathyroidism
c. Hyperphosphatemia
d. 1-25 dihydroxy Vitamin D3 deficiency
64. Patient diagnosed having renal cell carcinoma of upper pole of the left kidney. Before
doing surgery what should be done
a. MRI abdomen
b. Urinary catecholamine levels
c. Angiography
d. ………………

65. Factor associated with poor prognosis in FSGS is


a. Associated HTN
b. Heavy Proteinuria
c. Hematuria
d. ………………….

66. Anemia in CKD patient on erythropoietin therapy & allergic to for IV iron……..
a. Change iron sucrose or iron…
b. Increase EPO dose
c. ……………………
d. ……………………

67. Diabetic patient having ESRD, When to start renal replacement therapy
a. Before diagnosis of DM
b. GFR < 20
c. GFR <15
d. GFR <10
68. Cyclosporine
a. Henoch–Schönlein purpura

69. IV immunoglobulin pulse therapy


a. Humoral or Antibody mediated rejection

70. Plasmapheresis is used in treatment of


a. Antiglomerular basement membrane disease (Good pasture’s syndrome)

71. Cyclophosphamide used in treatment of


a. cANCA (Wegener’s granulomatosis)

72. Indomethacin
a. Type IV Renal tubular acidosis

73. Diarrhea
a. Normal anion gap metabolic acidosis

74. Methanol intoxication


a. High anion gap metabolic acidosis

75. What is the best immediate treatment to decrease the potassium levels
a. Ca gluconate
b. Ca resonium
c. Insulin & glucose infusion
d. NaHCO3
76. A patient with BP=80/50mmHg, dehydrated having Na=110. Which solution is best
treatment for the patient
a. Half normal saline
b. Colloids
c. Dextrose
d. Normal saline
e. Albumin

77. All of the following are present in post streptococcal GN except


a. Low CH50
b. Low C3
c. Properdine
d. High ASO titer
e. Low C4

78. What is the most important in treatment of uremic bleeding


a. Cryoprecipitate
b. FFPs
c. Conjugated estrogen
d. Desmopressin

79. Erythropoietin is increased in all of the following except


a. Polycystic kidney disease
b. Renal artery stenosis
c. Renal cell carcinoma
d. Nephrogenic DI
80. Treatment of non invasive transitional cell carcinoma of bladder is
a. Radiotherapy
b. Transluminal urethral resection of bladder tumor
c. Palliative radiotherapy
d. Radical cystectomy

81. What is the most predisposing cause of CA bladder


a. History of smoking & cyclophosphamide
b. _____________
c. _____________
d. _____________

82. A patient on HD for 7 years has pain in both hands started in antecubital fossa, radiating
to volar aspect of the hand & marked at night & early morning. On examination numbness
(thumb, index & middle fingers) & hand grip is normal. What is the most likely finding
a. Osteophytes in cervical vertebra C5-C6
b. Uremic osteodystrophy
c. β 2 microglobulinemic deposition in flexor retinaculum
d. Carpal tunnel syndrome

83. Best treatment of diabetic nephropathy with Proteinuria


a. Frusemide
b. Captopril
c. Dipyridamol
d. Nifedipine
84. What is the most useful diagnostic test to differentiate unilateral renal artery stenosis from
bilateral renal artery stenosis
a. B/L measurement of rennin from renal artery
b. Differential renin measurement from both veins
c. Captopril uptake test
d. Peripheral renin measurement

85. All of the following can be associated with normal or increased size kidneys on U/S
except
a. Amyloidosis
b. Diabetic nephropathy
c. Scleroderma
d. Nephrocalcinosis

86. All of the following can cause hypocalcemia except


a. Renal failure
b. Hypoparathyroidism
c. Pseudohypoparathyroidism
d. Hypomagnesemia
e. Grave’s disease

87. Ultrasound can be diagnostic in all of the following except


a. Stone
b. Wilm’s tumor
c. Acute tubular necrosis
d. Hydronephrosis
e. Polycystic kidneys
88. Obstructive uropathy can be present in U/S in all of the following except
a. Stones
b. Ectopic ureter
c. Diabetes insipidus
d. Urinary tuberculosis

89. Which of the following is most diagnostic of UTI


a. Leukocyte esterase +
b. Dipstick nitrate +
c. Increased WBCs in urine
d. Increased WBCs in blood
e. Dipstick esterase & nitrate +

90. A case of upper respiratory tract infection & hematuria within 2 days & normal BP. What
is the microscopic finding
a. Mesangial proliferation
b. IgA deposition in mesengium
c. Epithelial cell proliferation
d. All of the above

91. A case of Henoch–Schönlein purpura. History of acute gastroenteritis & purpuric rash
upper thighs & buttocks, proteinuria & hematuria. What is the treatment
a. Corticosteroids
b. Plasmapheresis
c. Hemodialysis
d. Supportive
92. All of the following drugs are nephrotoxic except
a. Paracetamol
b. Ibuprofen
c. Cisplatin
d. Cyclophosphamide
e. Cisapride

93. A case of hypokalemia with normal BP, (K=3) & HCO3=32. Urinary K is 10 mmol.
What is the cause
a. Diuretics
b. Vomiting
c. Hyperaldosteronism
d. Bartter syndrome
e. Corticosteroids

94. Healthy female 41y. Urine dipstick analysis ++ protein. No other pathology in urine.
What is the next valuable test
a. US abdomen
b. 24h urine for proteinuria
c. Urine Protein electrophoresis
d. Urine C/S
e. Urine analysis from first urine voiding

95. Most diagnostic test in acute glomerulonephritis


a. RBCs in urine
b. RBC casts
c. WBC casts
d. Dysmorphic RBCs
e. Tubular casts
96. NaHCO3 infusion is used in which of the high anion gap metabolic acidosis
a. Diabetic Ketoacidosis
b. Alcoholic intoxication
c. Acute kidney injury
d. Renal tubular acidosis

97. What is the most common preceding symptom of Alport’s syndrome


a. Microscopic hematuria
b. Microscopic Proteinuria
c. Both microscopic hematuria & Proteinuria
d. Conduction deafness
e. Macroscopic hematuria

98. All can cause acute nephritic syndrome except


a. Poststreptococcal GN
b. Mesengiocapillary GN
c. IgA nephropathy
d. …………………

99. A case of head trauma & urine output 300 ml/h, urine osmolarity < 600, and plasma
osmolarity 321. Which is the best treatment to replace urine output
a. Normal saline
b. Dextrose saline
c. Normal saline in dextrose
d. Colloids
e. Aqueous vasopressin
100. All are associated with renal osteodystrophy except
a. Hypocalcemia
b. Hyperphosphatemia
c. 1,25 dihydro cholecalciferol deficiency
d. Calcitonin +

101. All are present in minimal change glomerulonephritis


a. 90% responding to steroids
b. Age from 2-5 years
c. Low C4
d. Frequent relapse occur in 1st 4 years

102. All can cause hypercalcemia except


a. Primary hyperparathyroidism
b. 2ndry hyperparathyroidism
c. Tertiary hyperparathyroidism
d. Malignant metastasis
e. Sarcoidosis

103. Best solution to give in contrast nephropathy


a. Normal saline
b. D5%
c. Half normal saline
d. Crystalloids

104. Bartter syndrome


a. Increased renin & aldosterone
105. In renal failure, hypertension is due to
a. Increased ECF
b. Increased renin & aldosterone

106. Pheochromocytoma
a. Decreased ECF ???

107. Diabetes mellitus


a. Increased renin & aldosterone
b. Decreased renin & aldosterone

108. A patient with loss of skin turgor, obtundation & normal BP. Na= 160. You should give
dextrose with which of the following
a. Normal saline
b. Half normal saline
c. NaHCO3
d. Water 150 cc/h

109. Pneumococcal peritonitis is present in which of the following


a. Minimal change GN
b. Mesenteric adenitis
c. Acute appendicitis
d. Acute cholecystitis

110. Regarding prune belly syndrome


a. Other name Eagle Barrette Syndrome
b. Scoliosis is present
c. Intrauterine fetal death
d. Obstructed prostatic urethra
111. Diagnosed case of post streptococcal GN. What is the treatment
a. Corticosteroids
b. Penicillin
c. Aspirin
d. Supportive

112. A case of Wegener’s granulomatosis, hematuria & renal impairment. What is the
treatment
a. Supportive
b. Hemodialysis
c. Corticosteroids & cyclophosphamide
d. Plasmapheresis

113. Aspirin can cause all of the following except


a. Hyperglycemia
b. Hypokalemia
c. Hypercalcemia
d. Hemolytic uremic syndrome???

114. A case of repeated upper respiratory tract infection, hemoptysis with hematuria. O/E
bilateral edema & BP=145/95mmHg. CXR showing bilateral opacities. What is the diagnosis
a. Wegener’s granulomatosis
b. Antiglomerular basement membrane disease
c. IgA nephropathy
d. Glomerulonephritis
115. A case in ICU with flail chest on ventilator. ABGs --- pH=7.27, PCO2=30, HCO3=15.
What is the abnormality in ABGs
a. Metabolic acidosis
b. Metabolic alkalosis
c. Metabolic acidosis with respiratory acidosis
d. Metabolic alkalosis with respiratory alkalosis

116. A case of hyperkalemia with metabolic acidosis. All are true except
a. Alcoholic toxicity
b. Diabetic ketoacidosis
c. Aspirin toxicity
d. Bartter syndrome

117. Barter syndrome

118. ABGs

119. Anion Gap

120. If plasma osmolality is 330mosm/kg what happens to intracellualr & extracellualr


compartments

121. Salicylate Toxicity

122. Ethanol poisoning

123. Hypocalcemia

124. UTI in infancy & early childhood


125. UTI in Pregnancy

126. Calcium Oxalate stones (75%)

127. Pheochromocytoma

128. Renal artery stenosis (3-4 MCQs)

129. Renal artery stenosis diagnosis

130. What happens to the opposite normal unaffected kidney in Renal artery stenosis

131. What happens to the remaining kidney after nephrectomy

132. 2ndy hyperparathyroidism

133. Pre-Renal & ATN differences

134. Interstitial nephritis (Difference between NSAIDs & Antibiotics induced TIN)

135. Cholesterol Embolization

136. Waxy casts

137. IgA Nephropathy Vs Post-streptococcal

138. Post-Infectious Nephropathy

139. Henoch Schonlein Purpura


140. SLE treatment

141. AIDS (Ifosfamide in HIV)

142. CMV (2-3 MCQs)

143. Differences between cyclosporine & tacrolimus toxicity

144. Best treatment for Anemia in CKD

145. EPO Deficiency in CKD pathogenesis & effects

146. EPO increased in which conditions

147. ADPKD (2-3 MCQs)

148. Screening in ADPKD

149. Dialysis (4-5 MCQs)

150. How to remove chloramine from dialysis water

151. KT/V

152. Treatment of hypertension

153. Side effects of ACEi/ARBs

154. Hyponatremia
155. Hypokalemia ECG Changes

156. Hypokalemia Treatment

157. Hyperkalemia

158. Treatment of Ascites in CKD

159. Treatment of Calcium stones is normal calcium intake

160. Renal tubular acidosis

161. Causes of calcium oxalate crystals in urine

162. Multiple myeloma

163. Amyloidosis

164. Nephrotic syndrome

165. Analgesic Nephropathy

166. Nephritic syndrome

167. Which one is nephrotoxic


a. Paracetamol
b. Amphoterecin
c. ………………..

168. Mechanism of kidney injury by sulfamethoxazole


169. Transplant Rejection

170. Alport’s Syndrome (Recurrence after Renal Transplant)

171. Serum complement levels is not decreased in which conditions


a. Bacterial endocarditis
b. SLE
c. Post-infectious GN
d. IgA nephropathy

172. Complications of NSAIDs nephropathy

173. Hypernatremia

174. Hypo & Hyperphosphatemia

175. Commonest cause of death in CKD (infections & CV diseases)

176. What is the cause of calcification in HD


a. Hyperphosphatemia
b. Vitamin D intake
c. Increased Ca intake

177. Sterile pyuria

178. Renal Cell carcinoma

179. Treatment of bladder carcinoma involving muscle wall


180. Bladder carcinoma risk factors

181. BPH & urinary obstruction

182. Patient has DM has right flank pain & fever: USG shows gas in renal parenchyma
a. Emphysematous pyelonephritis
b. Xanthomatomas
c. Stone disease
d. Psoas abscess
e. Perinephric abscess

183. The most common renal tumor in children is:


a. Ureter tumor
b. Renal cell cancer
c. Transurethral cell cancer
d. Squamous cell cancer

**The most common cancer is Wilm’s tumor (nephroblastoma)

184. Vitamin D deficiency is associated with:


a. Osteomalacia
b. Adynamic bone disease
c. Osteoporosis
d. Osteopetrosis

185. Patient with DM on HD developed hypotension, echo is normal, what is the most likely
cause:
a. Inappropriate dry weight
b. Autonomic neuropathy
c. Dialysis reaction

186. Lithium is associated with:


a. Nephrogenic DI
b. Cranial DI
c. RTA type 2
d. Glomeulonephritis

187. Patient has lithium toxicity, what is the best treatment:


a. HD
b. PD
c. HDF
d. Plasmapheresis

188. Patient with salicylate poisoning has which type of abnormality:


a. Respiratory acidosis + metabolic acidosis
b. Respiratory alkalosis + metabolic acidosis
c. Respiratory alkalosis + metabolic alkalosis

189. Patient diagnosed with unilateral renal artery stenosis, the best next investigation:
a. MRA
b. CTA
c. DTPA scan
d. Captopril scan

190. The treatment of unilateral renal artery stenosis is:


a. ACEI
b. ARBs
c. Calcium channel blockers
d. Beta blockers

191. Which stone is radiolucent:


a. Calcium oxalate
b. Calcium phosphate
c. Cystine stone
d. Uric acid stone

192. Ifosfamide is associated with:


a. RTA 1
b. RTA 2
c. RTA 3
d. RTA 4

193. The best treatment of tumor lysis syndrome:


a. Hydration
b. Sodium bicarbonate
c. Hydration & N-acetyl cystein
d. Theophylline

194. DM is associated with:


a. RTA 1
b. RTA 2
c. RTA 4

195. Patient with DM started with ACEI, patient developed <20% rise in creatinine, what is
the next step:
a. DC ACEI
b. Continue ACEI
c. Start ARB
d. Patient has unilateral RAS

196. The best access for HD:


a. AVF
b. AVG
c. Permanent catheter
d. Temporary catheter

197. Patient initiated HD 2 weeks ago and developed DVT in leg & purpura, the most likely
diagnosis:
a. Heparin induced thrombocytopenia 1
b. Heparin induced thrombocytopenia 2
c. Hypercoagulopathy
d. DIC
**Clinical features of heparin-induced thrombocytopenia (HIT) that help distinguish it from
other forms of thrombocytopenia include the timing of onset (usually 5-14 d after beginning
an immunizing exposure to heparin), the severity of the thrombocytopenia (usually mild to
moderate), and the presence of thrombosis or other sequelae

198. SLE on HD is associated with:


a. Recurrent thrombosis
b. Poor quality of HD
c. Anti phospholipid syndrome
d. Graft failure

**SLE patients on HD have high risk for vascular access thrombosis


199. Recurrent peritonitis is associated with:
a. UF failure
b. Poor quality of dialysis
c. Poor control of blood pressure
d. Peripheral neuropathy

**Recurrent peritonitis may damage the peritoneum causing sclerosing encapsulating


peritonitis (SEP) leading to UF failure.

200. HDF is associated with:


a. Decrease mortality
b. Increase mortality
c. Increase mortality and morbidity
d. Poor quality of HD

**HDF is associated with decreased mortality and morbidity

201. The best treatment for methanol poisoning is:


a. HD
b. HDF
c. PD
d. CVVHDF

202. PD patient developed abdominal hernia, the next step is:


a. HD
b. Continue PD
c. Conservative treatment
d. HD and hernia repair

203. Autosomal dominant FSGS is associated with defect in:


a. Podocin
b. TRCP6

**AD FSGS caused by defect in ACTN4 encodes a membrane protein – α-actinin-4 ,TRCP6,
CD2AP
AR FSGS caused by defect in
NPHS1 encodes a membrane protein – nephrin
NPHS2 encodes a membrane protein – podocin

204. The most common cause of nephrotic syndrome in children is:


a. Minimal change disease
b. Membranous nephropathy
c. Mesengioproliferative GN
d. FSGS
e. IgA Nephropathy

205. Hepatitis C is associated with:


a. MPGN 1
b. MPGN 2
c. MNP
d. FSGS

206. The best treatment for anti-GBM along with steroids is:
a. Plasmapheresis
b. Hemodialysis
c. HD & PP
d. Conservative treatment

207. Patient with SLE has hypertension, hematuria, proteinuria& mild renal impairment.
What is the WHO classification?

**Stages are (WHO classification)


I – Minimal mesangial LN – mild, normal urine analysis and creatinine
II – Mesangial proliferative LN – HEMATURIA, PU, no HTN
III – Focal LN – HEMATURIA, PU, RI not progressive, HTN
IV – Diffuse LN – HEMATURIA, PU, RI, HTN
V – Membranous LN – can be clinically silent!!!
VI – Advanced sclerosing LN – slowly progressive RI, PU, bland urine sediment

208. Collapsing FSGS is associated with:


a. HIV
b. CMV
c. HCV
d. EBV
e. Heroin

209. Sickle cell disease is associated:


a. MPGN
b. MNP
c. Secondary FSGS
d. Cryoglobulin
**SCD causes (renal): renal infarction, renal papillary necrosis, RTA 2, ARF, renal
medullary carcinoma, UTI, HTN, MPGN

210. Patient has sepsis, multiorgan failure, hypotension, Renal Failure, the best management
is:
a. CVVHD
b. HD
c. PD
d. Conservative treatment

211. Patient on thiazide diuretics, what is most common:


a. Hyponatremia
b. Hypernatremia
c. Hyperkalemia
d. Hypermagnesemia

212. The best initial treatment of hypercalcemia is:


a. Hydration
b. Hydration & Frusemide
c. Bisphosphonates
d. Calcitonin

213. Patient with transplant has antibody mediated rejection, apart from steroids, the
treatment is:
a. Plasmapheresis
b. HD
c. Plasmapheresis & HD

214. The next best step in adult nephrotic syndrome:


a. Renal biopsy
b. Renal biopsy + steroids
c. Conservative treatment

215. Diarrhea is associated with:


a. High AG metabolic acidosis
b. Low anion gap metabolic acidosis
c. Normal Anion gap metabolic acidosis
d. Metabolic alkalosis & acidosis

216. The poor prognostic factors in primary membranous nephropathy are all except:
a. Female
b. Heavy proteinuria
c. HLA
d. HTN
** Many risk factors have been found, such as male sex, HLA type DR3+/B8+, white race,
advanced age, and tubulointerstitial changes or focal sclerosis found with renal biopsy. In
addition, nephrotic syndrome, elevation of immunoglobulin G excretion or beta2-
microglobulin excretion, low serum albumin, high serum cholesterol, an elevation of urinary
excretion of complement activation products, impaired renal function at diagnosis, and,
finally, hypertension are associated with a higher risk of renal function deterioration during
follow-up

217. Malignant HTN is associated with:


a. Vision loss
b. Acute LVF

218. What is the best access for HD?


a. AVF
b. AVG
c. Tenchkoff catheter
d. Temporary line

219. What is specific treatment for asymptomatic hematuria: (CWA)


a. Steroid
b. Immunosuppressive treatment
c. Plasmapheresis
d. None

220. Patient started on treatment for high BP, develops the following labs: Na 125, K 3.2, Ca
3.1, the most likely drug is:
a. Beta blockers
b. Thiazides
c. Calcium channel blockers
d. Frusemide

221. If ECF osmolality is 320, what would be the change on ICF:


a. Increase ICF & osm 320
b. Decrease ICF & osm < 290

**When ECF osm increases, larger flow from ICF to ECF occurs, resulting in decrease of
ICF and increase ICF osm

222. Analgesic nephropathy is associated with all except: (CWA) 507 ‫انظر‬
a. Malignant HTN
b. Transitional carcinoma of ureter
c. Small kidneys
d. Nephrocalcinosis

223. Small kidneys are hallmark of all except:


a. Analgesic nephropathy
b. CRF secondary to GN
c. Amyloidosis
d. Chronic pyelonephritis

224. Sterile pyuria is characteristic features of all except:


a. Acute interstitial nephritis
b. Renal Tuberculosis
c. Acute GN

225. 50 y/o patient with DM 2, BP > 140/95, the anti-hypertensive of choice is:
a. Calcium channel blockers
b. Beta blockers
c. ACEI
d. Hydralazine

226. CAPD is associated with:


a. Fluid overload
b. Protein loss
c. Glucose intolerance
d. All of the above
e. None of the above
227. Regarding glucose solution used in CAPD, the following statement is most appropriate:
a. Icodextrin use is more favorable the conventional glucose
b. 2.5% solution is better than 1.5% solution

228. Change of glucose from 1.5 to 4.25% solution in CAPD regimen, what you would
expect:
a. Decrease UF
b. Increase UF
c. More solute clearance

229. Patient with alcohol intake history, presented with coma and pH 7.23, labs???, he is
likely to have:
a. Alcoholic ketoacidosis
b. Non anion gap acidosis

230. Ifosfamide is associated with:


a. Renal tubular acidosis-2
b. Renal tubular acidosis-1
c. Renal tubular acidosis-4
d. Hypermagnesemia
e. Hyperkalemia

231. 45 years old lady on HD since 3 yrs, on cellulose acetate dialyzer, 3x per wk with 1.5m2
dialyzer, has Kt/V 1.42, what would you suggest:
a. Change to low flux dialyzer
b. Change to high flux dialyzer
c. Change to polysulfone dialyzer +
**Minimum of sp Kt/V > 1.2 is required for a 3x/wk dialysis, and a target of Kt/V >1.4,
therefore dialysis is adequate, and no need to change anything.

232. Patient with colonic carcinoma, presented with proteinuria, underwent renal biopsy, the
expected diagnosis:
a. FSGS
b. MCD
c. MNP

233. 50 years old male has RBC+ in urine dipstick, but microscopy for RBC is negative, the
cause is:
a. Hb in urine
b. High sp gravity of urine
c. High osmolarity of urine

234. 70 yrs old male with diarrhea, and volume loss, admitted in hospital, labs show: Na 170,
K 3.8, the management is:
a. Normal saline
b. D5 0.45S +
c. RL

235. 60 yrs female with h/o alcoholic over dose admitted to hospital, develops diarrhea, the
expected acid base changes:
a. Na—K—Cl—HCO3—pH levels
b. High AG metabolic acidosis +
c. Metabolic alkalosis
d. Metabolic acidosis + alkalosis
e. Normal AG metabolic acidosis
236. Renal agenesis is associated with:
a. Meckle’s diverticulum
b. Nephroblastoma

**These abnormalities include a variety of syndromes: Chromosomal anomalies, Caudal


dysgenesis, VATERL, and isolated anomalies of the cardiovascular, skeletal, and central
nervous systems.
Caudal dysplasia syndrome (CDS), Sirenomelia
Unilateral renal agenesis – VUR, UVJO, UPJO, structural malformations of the heart (eg,
septal defects), gastrointestinal tract (eg, anal atresia), genital, or skeletal systems.

237. FSGS AR type is associated with gene mutation:


a. Nephrin
b. Podocin
c. Alpha actinin 4

238. 34 yrs old female, BP 174/112, Na-139, K-3.4, metabolic alkalosis, renal US normal for
RAS, next investigation is:
a. Cortisol level
b. Aldoterone:renin ratio
239. 30 yrs old female, BP 180/110, attacks of nervousness, next investigation is:
a. Aldoterone:renin ratio
b. Urinary catecholamines
c. CT scan abdomen

240. Patient with unilateral RAS, diagnostic method: (CWA)


a. MRA
b. CTA
c. MRI abdomen + gadolinium
d. Diff renin activity in renal vein
e. Diff renin activity in renal artery

241. Renal Transplantation patient with pharyngitis, all can be prescribed except:
a. Augmentin
b. Cefazolin
c. Erythromycin

242. Following drugs need to increase the dose of CNI if used: except
a. Phenytoin
b. INH
c. Carbamezepine
d. Rifamicin
**INH will decrease CNI level

243. 36 yrs old male with family h/o ADPCKD, BP 110/70, no proteinuria, what
investigation to order:
a. CT scan abdomen
b. Genetic study
c. US abdomen ++
244. Patient with calcium oxalate stone, you advise for:
a. Low Ca diet intake
b. High Ca diet
c. High Na diet

245. All of the following can cause acute interstitial nephritis except:
a. Paracetamol
b. Rifampicin
c. NSAID

246. Patient with HD for 8 yrs with shoulder & bone pains, cute interstitial nephritis, B2
amyloidosis is provisional diagnosis, to confirm the test is:
a. X rays bones
b. Amyloid component in…**
…………………..
**The Amyloid found in bone cysts is different, but the B2M levels are not indicative

247. 30 yrs old female with 14 wks gestational amenorrhea and mild proteinuria, the option
for anti hypertensive is:
a. ACEI
b. Thiazides
c. Labetolol
248. Regarding UTI in pregnancy, most correct statement is:
a. Acute pyelonephritis occurs in 1-2% of all pregnancies
b. Pyelonephritis is very common
c. 50% will develop asymptomatic bacteruria

**2-10% will develop asymptomatic bacteruria,of these patients more than 40% will develop
pyelonephritisoverall pyelonephritis is 1-2% of all pregnancies.

249. 24 yrs old female with microscopic hematuria, normal RFT, h/o grandfather on HD,
uncle with CKD and deafness, likely diagnosis is:
a. ADPCKD
b. ARPCKD
c. Alport’s syndrome

250. 25 yrs old male with microscopic hematuria, undergoes biopsy – normal on LM, most
likely diagnosis:
a. MNP
b. FSGS
c. Thin basement membrane disease

251. Bad prognostic factor in MNP:


a. HTN
b. RI
c. Heavy proteinuria for 6 months
252. An alcoholic man with diarrhea & vomiting, came to ER with hemolysis and
rhabdomyolysis, respiratory depression, what abnormality expected:
a. Hyperphosphatemia
b. Hypophosphatemia
c. Hypomagnesemia

**Hemolysis & rhabdomyolysis will cause hyperPO4


Diarrhea & alcoholism can cause hypoPO4
HypoPO4 can lead to respiratory depression and it can be caused by resp alkalosis
Severe hypoPO4 will lead to rhabdomyolysis

253. 40 yrs old male underwent live related renal transplantation, one month later presents
with gastritis, vomiting, diarrhea, diagnosed with CMV disease, to confirm:
a. CMV Ab
b. CMV Ag
c. Stool culture
d. US

254. Absolute contraindication for living kidney donation is:


a. HTN
b. GFR < 80 +
c. Age > 60

**Absolute CI: psychiatric disorder, renal disease, abnormal renal anatomy, stones, DM,
HTN, IHD, neoplasm, malignancy, infection, CLD, neurological disease, anticoagulation
use, pregnancy, thrombotic disease
Relative CI: ABO incompatibility, Age <18 or >65, mild HTN, single episode of stones,
smoking
255. Manifestation of hyperkalemia on ECG:
a. Tall peaked T wave

256. Hyperkalemic manifestation:


a. Ascending paralysis

257. Patient with Na 160, fluid intake is:


a. Dextrose 5%L

258. Patient with symptomatic hypovolemic Na 116, you should give him:
a. Hypertonic saline

259. Patient with Na 162 on NGT, the cause:


a. NG suction

**NG suction will cause vomiting, causing hypokalemia and hyponatremia, maybe if severely
dehydrated will cause hypernatremia

260. Patient with general weakness, low K, low HCO3:


a. Bartter syndrome

261. Patient with euvolemic hyponatremia:


a. SIADH

262. Most common cause of post-RTX for delayed graft function is:
a. ATN

263. Most common cause of nephrotic syndrome in children is:


a. MCD
264. Drugs that increase Cyclosporine level:
a. Fluconazole

265. Pregnant woman with hypertension, impaired RFT:


a. Pre-eclampsia

266. Patient on NSAID, impaired RFT:


a. Acute Interstitial Nephritis

267. Patient with SLE, lupus cerebritis, not responding to cyclophosphamide and steroids:
a. Plasmapheresis

268. Child with nephrotic syndrome not responding to steroids:


a. Cyclosporine

269. Patient with IgA NP, biopsy will show:


a. Mesangial proliferation

270. 40 years old male with nephrotic syndrome, the most common cause is:
a. Membranous nephropathy

271. Patient with rhabdomyolysis, the treatment is:


a. Fluids

272. The most common cause of hypercalcemia, Nephrocalcinosis is:


a. RTA-1
**The most common cause of hypercalcemia and nephrocalcinosis, is primary
hyperparathyroidism, and the second cause is distal RTA (type 1)
273. Alport’s syndrome, bad prognosis is:
a. X-linked male

**X-linked is > 80%, females are less affected than males


AR is 15%, females are affected like males
AD is 5%

274. Patient with dialysis with UF failure:


a. Shift to nocturnal APD

275. Patient with peritonitis, diagnosed by:


a. Pain & culture (+)

276. Patient with high PTH on HD, hyperphosphatemia:


a. Secondary hyperparathyroidism

277. Pregnant lady, 3rd trimester, admitted with HTN, proteinuria, lower limb edema (? Pre-
eclampsia) BP 160/110, what is next?
a. Give oral hydralazine then continue on labetalol to DBP < 90
b. IV enalapril
c. IV nitroprusside

278. Young male with episodic hypertension & nervousness, next step is:
a. 24 hours urine collection for catecholamines

279. In HD, chloramine is removed by:


a. Carbon adsorption
b. Reverse osmosis
280. Dialysis patient, always HTN, for the last few months has pre-dialysis pressure of
100/50, what’s next step:
a. Do echo

281. DM, ESRD, has failed AVF on both upper limbs, what is the next step:
a. Perm catheter
b. CAPD
c. Kidney transplant
d. Observation

282. CAPD has the following complications:


a. Fluid overload
b. Hyperglycemia
c. Protein loss
d. All of the above

283. Patient on HD, single pool Kt/V 1.2, what’s the next step:
a. Keep on same treatment

284. CVVHD is the best modality for the following patients:


a. Uremic pericarditis
b. Hepatic encephalopathy +
c. HTN with encephalopathy
d. Hyperkalemia with renal failure
e. Pulmonary edema
285. Patient with severe hyperkalemia K = 7.9, next step:
a. Ca gluconate

286. Trimethoprim can cause high serum creatinine by:


a. Block epithelial Na channel a? c?
b. Decrease aldosterone secretion
c. Inhibiting tubular secretion of creatinine +

287. Which of the following ABGs you can fit for patient with RA on and salicylate toxicity:
a. pH 7.3 – HCO3 16 – PCO2 22

288. A young man with suspicion of ethyl glycol toxicity, has serum Osm 330, Na 140,
normal renal function, HCO3 22, no crystals in urine, nausea but no vomiting, what’s next:
a. Give fomepizole
b. Follow up because this is mostly ethanol toxicity

**Fomepizole is given in this case, it is more superior to ethanol, and HD is the last resort
Fomepizole has replace ethanol (due to CNS manifestation)
If ingestion is since 2 – 3 hours, then gastric lavage with charcoal should be performed.l

289. Patient with K – 4, non anion gap metabolic acidosis, positive urine anion gap, what is
the diagnosis
a. Distal RTA
b. Acetazolamide
c. Diarrhea
**Acetazolamide cause non anion gap metabolic acidosis. RTA the same but with hyper or
hypoK
290. In which of the following condition with high AG metabolic acidosis, NaHCO3 shows
decrease mortality:
a. Septic shock
b. Alcoholic ketoacidosis
c. Acute kidney injury
d. Ethanol toxicity
e. Heroin toxicity

291. Plasma volume is:


a. 2/3 ECF
b. Minimal amount of ECF
c. 1/3 ECF

292. Female on OCP, has polyuria and polydipsia, Urine osmolality 160, Na 133, sugar
normal, the most likely diagnosis:
a. NDI
b. CDI
c. DM
d. Psychogenic DI

**From U osm, she has DI, but type is according to WDT and ADH response
If U osm increase to 750 after ADH --- CDI
If no change --- NDI
If U osm increase to 750 after WDT --- Polydipsia
Answer is [hide]????….[hide]
293. Most common kidney stone:
a. Ca oxalate

294. Patient presents with picture of renal colic, next step:


a. CT scan w/o contrast

295. Main site of reabsorption of Mg+2:


a. Thick loop of Henle

296. Young boy with hematuria, his sister also has hematuria, (? Suggestive of TBMD),
what’s next:
a. Renal biopsy , EM for GBM

297. Young patient with hematuria, h/o stone disease, father is ESRD, most likely diagnosis
is:
a. ADPKD
b. Alport’s syndrome
c. Thin basement membrane disease

298. Patient has oliguria, muddy brown casts:


a. ATN
b. Acute interstitial nephritis
c. GN

299. Patient with Burkett’s lymphoma, started allopurinol, he presented with picture of tumor
lysis syndrome:
a. Frusemide
b. PD
c. HD
300. ADPKD is mostly associated with cyst in:
a. Liver
b. Ovary
c. Mesenteric

301. Which of the following is diagnostic of acute interstitial nephritis:


a. Renal biopsy
b. Sterile pyuria
c. WBC casts

302. HIV patient on ceftazidime and sulfamethaxazole, serum creatinine is rising, the cause
is:
a. Acute interstitial nephritis

303. In analgesic nephropathy, all can happen except: (CWA)


a. Papillary necrosis
b. Nephrocalcinosis
c. Malignant HTN

304. All of the following will slow progression of CKD except:


a. High protein diet

305. One is characteristic urine findings in ESRD: (CWA)


a. Waxy cast
b. Hyaline cast
c. RBC cast
d. WBC cast
306. CKD patient with generalized bone pains (PTH, Ca, PO4, ALP) all normal:
a. Adynamic bone disease

307. All of the following are characterized by low complement except: (CWA)
a. IgA nephropathy

308. 20 years old patient DM 1, normal RFT, GFR 90, microalbuminuria, what is the risk of
pregnancy on renal function:
a. No risk
b. RFT will deteriorate during pregnancy the improve after deliver

309. One of the following is not characteristic of Henoch Schonlein purpura:


a. Splenomegaly
b. Palpable non TPP
c. Arthritis
d. GI involvement

310. All cause of RPGN except:


a. Lupus nephritis
b. IgA nephropathy
c. Membranous nephropathy
d. Poststreptococcal GN
e. Goodpasture's syndrome

311. Patient with hematuria, biopsy LM showed mesangioproliferation, EM showed deposits


in mesangium:
a. IgA NP
312. High BP, hematuria, AKI, RBC casts:
a. Acute GN

313. Post strep GN is cause by:


a. Group A strep

314. High Ca, low K, low Na:


a. Thiazide

315. Mechanism of amiloride:


a. Block epithelial Na channel in cortical collecting tubules & ducts

316. Drug affecting Cr without affecting GFR:


a. Cimetidine

317. HTN patient, started of ACEI, Cr increased to 115, what is next:


a. Reassure

318. One of the following is not nephrotoxic:


a. Paracetamol
b. Cyclosporine
c. Lithium
d. Cisplatin

319. 12 years old, proteiuria 2+, normal Cr, no hematuria, what’s next:
a. Early morning dipstick
320. 60 years male, heavy smoker, alcohol abuser, hematuria, > 90% normomorphic RBC,
what’s next:
a. Abdomen CT scan

321. In rhabdomyolysis, all except:


a. Hypercalcemia

322. Elderly patient with atherosclerosis, AAA, underwent repair, developed AKI, biopsy
showed needle shape crystals:
a. Cholesterol emboli

323. Which is true regarding cholesterol emboli:


a. Livido reticularis
b. Skin nodule
c. Eosinophilia
d. All of the above

324. Patient with nephrotic syndrome developed ESRD after 2 years (no biopsy), underwent
1st renal transplantation, failed after 2 months, then did 2nd renal transplantation, 6 month
later developed nephrotic syndrome, what is the most likely cause:
a. FSGS

325. Renal transplantation with h/o hyperlipidemia, she cannot tolerate statin, has Hirsutism,
the best immunosuppressant:
a. MMF + TAC
b. mTOR + TAC
c. mTOR + AZA
326. MMF in comparison with AZA:
a. Less episodes of acute rejection

327. Potential kidney donor, isolated hematuria, normal creatinine, no proteinuria, no UTI, no
stones, what to do:
a. IVP
b. Cystoscopy and CT abdomen, if normal proceed for transplant

328. A CKD patient with PTH 350, Ca 2.63, PO4 2.5, ALP 594, what is the treatment?
a. Calcitriol
b. Ca acetate
c. Sevelamer
d. None of the above

329. In infancy, oliguria is defined as: (CWA)


a. 0.1 ml/kg/hr
b. 0.3
c. 0.5
d. 1

**Infants <1ml/kg/hr, children <0.5, Adult <400


330. Patient with MI has cardiac catheterization. 3 weeks later he presented with painful blue
toes and fatigue, the diagnosis is:
a. Cholesterol emboli
b. Contrast induced nephropathy
c. Rhabdomyolysis
d. Tumor lysis syndrome
331. Cisplatin induced nephrotoxicity can be prevented by:
a. Hydration
b. Mg replacement
c. Forced diuresis

332. Rhabdomyolysis associated with all except:


a. Hypercalcemia

333. ESRD & DM patient has failure of AVF on both arms, what’s next:
a. Perm catheter
b. AVG
c. Conservative
d. CAPD

334. All associated with low compliment except:


a. IgA nephropathy

335. Patient with SLE, has DPGN active, but she is concerned about sterility, what’s next:
a. MMF
b. AZA
c. Plasmapheresis

**If a female patient asks about pregnancy then give her AZA, if male patient asks about
sterility then give MMF
336. What will 0.9% NS do:
a. Decrease osmolality
b. No change in osmolality
c. Will decrease osmolality first then increase

**In mammals, normal saline is osmotically balance

337. All associated with large kidneys except:


a. Analgesic nephropathy

338. Mechanism of action for amiloride


a. Blocks epithelial Na channels in collecting tubules

339. Mechanism of action of TMP/SMX, how it increase creatinine

340. Patient with DM 15yrs, diabetic nephropathy, diabetic retinopathy, BP 150/90, what is
the best anti hypertensive Rx:
a. ACEI
b. Beta blockers
c. Thiazide diuretics

341. 7yr child has Hypertension, proteinuria, hematuria, what is the diagnosis:
a. Postinfectious glomerulonephritis
b. IgA nephropathy
c. Mesengiopriliferative GN
d. Membranous nephropathy
342. Indication to start HD in diabetic
a. GFR < 10
b. GFR < 15
c. GFR < 20

343. DM & CKD with Cr 40, GFR 20, next step:


a. Vascular access
b. Start HD
c. Wait for uremic manifestation

344. Gold standard to measure GFR


a. Inulin
b. Isothalamate
c. DTPA scan

**DMSA scan is for morphology and structure

345. 31 years old male, episodic HTN, next step:


a. 24 hrs urine collection for catecholamines

346. Ultrasound will show all the following abnormalities except:


a. Medullary dysplastic kidney
b. Renal stone
c. ATN
d. Acute pyelonephritis
347. DM on ACEI, metformin, glipizide, HCO3 15, glucose normal, the likely cause of
metabolic acidosis:
a. DKA
b. Alcohol
c. Drug induced

348. A Cadaveric renal transplantation in 3 months develop fever, cough, loose motion, Hb <
9, PLT 100, TLC 3.2, next:
a. CMV Ab
b. CMV Ag
c. Stool culture
d. CXR

349. The most transmittable donor disease:


a. CMV
b. EBV
c. HBV
d. HCV

350. Renal transplantation patient in 2nd month, develop fever, cough, CXR bilateral
infiltration, the diagnosis is:
a. PCP
b. CMV
c. Fungal
d. None of the above

**PCP is usually 6 months post transplant


351. Patient with CKD and DM, develop MI, for coronary angiography, what is the next
treatment / precaution for contrast induced nephropathy:
a. Hydration
b. Theoplylline
c. Adenosisn
d. NaHCO3
e. None of the above

352. Antimicrobial and NSAID cause tubulointerstitial, the difference is:


a. Acidification defect in Antibiotics, concentrating defect in NSAID
b. Recovery is different
c. Fever, arthralgia, rash present in 20% of NSAID and 80% in antibiotics

353. The difference between tacrolimus & cyclosporine:


a. Neurotoxicity
b. Less diabetogenic
c. Cause gingival hypertrophy

354. Patient with rheumatoid arthritis on aspirin, the acid base is:
a. Respiratory alkalosis + metabolic acidosis

355. Best antibiotics for asymptomatic UTI in pregnancy:


a. Nitofurantoin
b. Ciprofloxacin
c. Ampicillin
d. Amoxicillin
e. TMP/SMX
356. CKD is classified according to:
a. GFR

357. 40 y/o age, h/o smoking and analgesia, has hematuria, cystoscopy normal, RBC
isomorphic, the next step is:
a. CT scan + contrast
b. Renal biopsy

358. What shows the glomerular origin of hematuria:


a. RBC casts
b. WBC casts

359. The characteristic findings in CKD:


a. Hypo Ca

360. ESRD on HD has normal size kidneys in all of the following except:
a. Analgesic nephropathy
b. Sarcoidosis
c. ADCKD
d. Amyloidosis

361. Following water deprivation test, and ADH, Plasma osmolaity 295, Urine osmolaity
760, Urine osmolality after ADH 780
a. Central diabetes inspidus
b. Nephrogenic diabetes inspidus
c. Polydipsia
362. 40 yrs, previously healthy, passes urine 20-30 times/day, glucose normal, Na 143, U Na
20, the diagnosis is:
a. CDI
b. NDI
c. Polydipsia
d. None of the above

363. Patient with lung carcinoma, labs show urinary Na 29, plasma Na 129, plasma
osmolality 258, urinary osmolality < 100, thyroid function tests normal, what is the treatment:
a. Fluid restriction

364. 32 years female, weakness, deny diuretic abuse, vomiting, excessive urine, urinary Na
59, normotensive, urinary potassium 30, HCO3 32, what is the diagnosis:
a. Diuretics
b. Diarrhea
c. Vomiting

365. Unilateral RAS, the next investigation:


a. Differential renal venous renin level

366. The most common renal stones:


a. Calcium oxalate

367. The most common shiga toxin:


a. E. coli 157
b. Salmonella
c. Shigella
d. None of the above
368. MNP is cause by which of the following drug?
a. Penicillamine
b. Vincristine

369. Patient with DM, HTN, Cr 268, K 5.5 – 5.9, the best antihypertensive drug is:
a. ACEI
b. ARBs
c. Frusemide
d. Amiloride
e. Propranalol

370. Patient with head trauma, brain hemorrhage, with hypernatremia, polyuria, the diagnosis
is:
a. DM
b. DI

371. Lady with DM, mild proteinuria on ACEI, asking about pregnancy. What is the effect of
pregnancy on her KFT.
a. No effect
b. Will worsen along pregnancy
c. KFT will improve

372. Most common cause of hypernatremia and polyuria is:


a. DM
b. DI
373. What is the percentage of asymptomatic hematuria in children?

374. Most common cause of seizures in hematuria is:


a. Hyperkalemia
b. Hypocalcemia
c. Hyponatremia
d. Increase FDP

375. Best method for controlling bleeding diathesis in HD is:


a. Cryoprecipitate
b. DDAVP
c. Platelets
d. Estrogen

376. 80 years old male with MI creatinine 80, urea 2.5, what is the cause of his labs:
a. Old age
b. Ischemia
c. DM
d. Dehydration

377. 63 years old male with asymptomatic hematuria wants to donate his kidney to his wife,
KFT normal, urine – no proteinuria, what is your decision:
a. Exclude from donation
b. Proceed with donation
c. Proceed after ruling out any malignancy or glomerular disease.
378. Skin manifestations of atheroembolic disease?

379. What are the ECG changes in hyperkalemia?

380. Causes of low compliments except:


a. Shunt nephritis
b. MPGN
c. SBE
d. SLE
e. IgA nephropathy

381. All can cause acute interstitial nephritis except:


a. Azathioprine
b. Cyclosporin
c. Lithium
d. NSAID

382. In unilateral RAS, what happens with the other kidney?


a. Increase renin
b. Increase aldosteron
c. Decrease renin

**In the contralateral kidney, the rennin & RBF decrease, I-131 & I-125 decrease, and it all
improves after revascularization of the stenotic kidny
383. ESRD & DM patient has failure of AVF on both arms, what’s the most likely option for
this patient:
a. Perm catheter
b. AVG
c. Conservative
d. CAPD
e. Transplant

384. The most common renal stones:


a. Ca oxalate

385. The most common shiga toxin:


a. E coli 157
b. Salmonella
c. Shigella
d. None of the above

386. Patient with renal colic, KUB free, IVP filling defects, the cause is:
a. Uric acid stone

387. Increase of all of the following will cause stones except:


a. Calcium
b. Citrate
c. Oxalate
d. Uric acid
e. Phosphate

388. Manifestations of distal RTA?


389. Patient with suprapubic pain, dysuria, KUB free, mild hematuria, no crystals, expected
to find on urine analysis:
a. Calcium oxalate
b. Nitrates + leukocytes

390. Renal transplantation patient in 4th month, develop fever, cough, CXR bilateral
infiltration, LDH, AST, ALT are high, the diagnosis is:
a. PCP
b. CMV
c. Fungal
d. None of the above

**PCP is usually 6 months post transplant, but LDH & LFT are high

391. High Ca, low K, low Na:


a. Thiazides

392. Mechanism of amiloride:


a. Block epithelial Na channel in cortical collecting tubules

393. Drug affecting Cr without affecting GFR:


a. Cimetidine
394. All cause of RPGN except:
a. Lupus nephritis
b. IgA nephropathy
c. Membranous nephropathy
d. Post infectious glomerulonephritis
e. Goodpasture’s syndrome

395. Waxy casts are:


a. Never found in normal urine
b. Can be found in normal persons
c. Present in acute interstitial nephritis
**Always found in CKD/ESRD, never found in normal urine analysis

396. Patient with abdomen pain, US showed gas around kidney, how to treat?
a. Emergency nephrectomy
b. Antibiotics
c. Observe
d. Percutaneous drainage

**The diagnosis is emphysematous pyelonephritis

397. 7 y/o male with hematuria, the main common cause is:
a. IgAN
b. Mesangioproliferative GN
c. Alport’s syndrome
d. MCD
e. FSGS
398. 7 y/o male with asymptomatic hematuria, father has the same issue, no proteinuria,
brother has the same issue, work up is negative, the diagnosis is:
a. Benign familial hematuria**

399. TMP/SMX can cause hyperkalemia in patients with AIDS:

**Reduce renal potassium excretion through the competitive inhibition of epithelial sodium
channels in the distal nephron, in a manner identical to the potassium-sparing diuretic
amiloride

400. Pneumococcal peritonitis is commonly found in:


a. Minmal change disease

**Common in children, rare in adults, due to cirrhosis, nephrotic syndrome, CAPD, BMT,
RA, SLE, genital tract infection in women

401. UTI is commonly due to:


a. Enterocoocus
b. Enterobacter
c. E. coli
d. Pseudomonas
402. Patient on CAPD, with bone pain, this is mainly due to:
a. Adynamic bone disease
b. Osteitis fibrosa cystica
c. Osteoporosis

403. Patient on CAPD, on 2.5 x 2 exchanges, 4.25 x 2 exchanges, with SOB and LL edema.
PET given Cr = 0.92, Ur = 1, D4/D0 S = 0.2 Next step is:
a. Change to nocturnal exchange 10 – 12 hrs
b. Change to 4.25 x 4 exchanges

**PET Test
**APD also associated with:
Lower daily sodium removal. (worse volume and BP control )
Rapid loss of residual renal function.
Higher protein losses with multiple night time exchanges.
More expensive
Less peritonitis

404. Patient with shortness of breath, CXR showed infiltrates, normal sinus X-ray, then
developed hemoptysis, rise in creatinine, next step is:
a. Lung biopsy
b. ANCA
c. Compliments
**The case goes with Goodpasteurs disease, and the diagnosis is done by anti-GBM levels,
or tissue biopsy from renals (linear stacute interstitial nephritising of the basement
membrane secondary to immunoglobulin G deposition) or lung (extensive hemorrhage with
accumulation of hemosiderin-laden macrophages within alveolar spaces)
cANCA can be positive in GPS and may precede the anti-GBM.

405. Pregnancy and bacteruria, all true except:


a. Screen all pregnant women for symptomatic bacteruria
b. No treatment required if asymptomatic
c. Dysuria can be to non urinary causes

406. The difference between tacrolimus & cyclosporine is:


a. Hirsutism
b. Hepatotoxic
c. Nephrotoxic
d. Neurotoxic

407. Common cause of PD peritonitis:


a. S. epidermidis
b. S. aureus
c. C. albicans
d. S. typhi
408. The benefit of dialyzer re-use is:

**The reuse of dialyzers is associated with


environmental contamination, allergic reactions, residual chemical infusion (rebound
release), inadequate concentration of disinfectants, and pyrogen reactions
Bleach used during reprocessing causes a progressive increase in dialyzer permeability to
larger molecules, including albumin
No differences in mortality between reusing and those not reusing dialyzers
The cost savings associated with reuse is substantial, especially with more expensive, high-
flux synthetic membrane dialyzers
Waste disposal is definitely decreased with the reuse of dialyzers, thematurias environmental
impacts are lessened

409. 12 y/o boy with general weakness and hyperkalemia, all of the following may be the
cause except:
a. Addison disease
b. Bartter syndrome
c. Gordon disease
d. SLE

410. All the following causes hyponatremia except:


a. Chloropramide
b. Vincristine
c. Lithium

411. 31 y/o male had a car accident and sustained a head trauma, the urine output is 300cc/hr
and Na = 170, the next step is:
a. D5W
b. Aqueous vasopressin
c. NS
d. Water restriction
e. Replace urine output

412. A question about prune-belly syndrome

A partial or complete lack of abdominal muscles. There may be wrinkly folds of skin covering
the abdomen. Undescended testicles in males. Urinary tract abnormality such as unusually
large ureters, distended bladder, accumulation and backflow of urine from the bladder to the
ureters and the kidneys Frequent urinary tract infections due to the inability to properly expel
urine. Later in life, a common symptom is post-ejaculatory discomfort. Most likely a bladder
spasm, it lasts about two hours.

413. Patient is scheduled for cisplatin dose, the best preventive method to avoid
nephrotoxicity is:
a. No particular prevention from cisplatin
b. Give Mg supplements pre dose
c. Hydration pre and post CTX dose

414. Patient underwent chemotherapy dose, and took hydration before the dose, but did not
take allopurinol before the dose. Now no urine output, Cr 400, uric acid very high, what to do
next?
a. Continue hydration and monitor KFT
b. Start giving allopurinol
c. Start HD

415. Lithium – NDI, RTA 1, acute kidney injury, minimal change disease, FSGS, CIN
416. Isolate hypodipsia – Uosm > 700

417. UTI in pregnancy – amoxicillin, nutrofurantoin, cephalexin

418. Preeclampsia – hospitalization and rest, if HELLP then delivery

DR. ASAD EXAM


419. Infusion of normal saline to healthy person results in:
a. ECF increase osmolarity & ICF decrease.
b. ECF decrease osmolarity & ICF increase.
c. No change in osmolarity
420. Pt. with severe respiratory infection C&S Pseudomonas. Sensitive to Gentamycin which
was given for 7 days, symptoms improved but lab result creatinine 450, was normal,
previously, urine free what dx:
a. Gentamycin toxicity
b. Interstitial nephritis
c. Volume depletion
d. ……………….

421. Pt in ICU sepsis receive nafcillin, vancomycin, creatinine 90, after 10 days creatinine
185, urine WBC, +protein. FeNa >2%, the cause of AKI is:
a. ATN
b. Interstitial nephritis
c. Papillary necrosis

422. Pt CKD on PD. CAPD 2l 2.5%&4.5% 2l , cr450, p/p0.9, serum sugar & albumin
normal, pt. developed lower limb edema best Rx:
a. Change to 4.5% 3l X4.
b. Change to nocturnal PD 10-12 cycles

423. Pt. ESRD on HD 3/WK develop ascites, not improving with dialysis…LFT normal,
serum albumin 3mg, best Rx:
a. Increase freq. of dialysis.
b. TIPPS
c. Leeven shunt.
d. Change to CAPD
424. Patient on PD, Develops 1st time peritonitis, fever, abdominal pain, turbid color fluid
aspirate. +culture staph aureus, received antibiotic treatment, condition improved. Next step:
a. CT abdomen
b. U/S for tunnel
c. No other diagnostic test

425. Pt with chronic hyponatremia, what you suspect:


a. Shift of electrolytes & minerals from brain out side
b. Brain produces new organic osmolytes with no shift.
c. Change pH intracellular.

426. 7 year old child on PD .C/O peritonitis. found to have fungal inf. Received flucanazol
according to culture but no response after 2wk Rx. Second choice:
a. Wait 2wk then resume PD dialysis.
b. Change to itracanazol
c. Remove catheter & start HD till treated

427. Pt.75 year old C/O chest pain at catheter, found to have urea 2.5, creatinine 79 mmol. lab
result due to:
a. Dehydration
b. Old age
c. Low protein diet

428. A 35 year old female previously well, not HTN or DM, take only OCC pill, vitamins,
c/o 20-30 time polyuria, Na 143, sugar normal, creatinine normal, urine osmolarity.>160 the
cause is:
a. DI.
b. NDI
c. Phy. polydypsia.
429. Pt DM, on HD failed his fistula both arm next step:
a. Graft.
b. Change to PD.
c. Perm catheter

430. Pt do IVP found papillary necrosis, cupping, outer scar suspect:


a. Analgesic nephropathy
b. Bilateral cortical necrosis
c. Acute interstitial nephritis

431. All cause normal size kidney in CKD except:


a. Sarcoidosis
b. Scleroderma
c. Nephrosclerosis

432. A 5-year-old child has Steroid resistant nephritic syndrome. The most common cause:
a. Membranous nephropathy
b. Minimal change disease
c. FSGS

433. Children with MCN on cyclosporine what unlikely S/E:


a. Hirsutism
b. Gingival hypertrophy
c. DM

434. A 37 years old male came for investigations, all was normal including urine analysis.
His father and uncle are on HD because of ADPKD. What next step:
a. CT scan
b. IVP
c. Renal U/S
d. Genetic testing

435. A 17-year-old male brought by family after high dose salicylic acid starts with vomiting
nausea. Then coma best Rx:
a. Infusion NaHCO3
b. Charcoal
c. HD

436. Old age male with history of decrease oral intake, Na172, spot urinary Na 10, normal
BP. Best Rx.:
a. DW5%
b. DW5% +.45% NS
c. NS.9

437. Obese pt having DM, HTN, complaining abdominal pain, vomiting & diarrhea. He is on
Rx enalapril, metformin, atenolol. pH low, HCO3 low, Na140, Chloride low.
{high anion gap metabolic acidosis] the cause:
a. Dehydration
b. Drug induced

438. Best to prevent AKI for pt. going for catheter whose c/o chest pain, DM, creatinine 170:
a. Mannitol
b. Frusemide
c. NS.9%

439. Hyponatremia may be present in all conditions except:


a. Increase water & decrease Na.
b. Decrease water & increase water.
c. Increase water & increase Na
d. Increase Frusemide 80-120 mg. increase hyponatremia

440. Pt. IHD, CABG, Cr 150, dry skin, urinary spot Na<10. Best to decrease renal failure:
a. Frusemide
b. Theophylline
c. Volume expansion

441. Pt normotensive K=3, HCO3=32, urinary K> 30. This is related to:
a. Barters syndrome
b. GI losses.

442. Alcoholic pt came to ER with severe diarrhea & recurrent vomiting. He is hypotensive,
tachycardia. [lab ABGS……pH low, normal anion gap]the acid base is:
a. Non anion gap metabolic acidosis.

443. Pt alcoholic, confused, vomiting, pH7.2, HCO3 16, PCO2 22, his ABGS:
a. High anion gap metabolic Acidosis + Respiratory alkalosis

444. All CKD With small kidney size except:


a. Amyloidosis
445. Kidney transp. Recipient live related donor, found to have fever & SOB, suspect CMV
best diagnosis is:
a. CMV IgM.
b. CMV IgG.
c. CMV Ag.
d. CMV PCR.

446. All the following proteins decrease in nephritic syndrome except:


a. Albumin
b. Fibrinogen
c. Ceruloplasmin
d. Ferritin

447. Patient has DM on losartan for 36 month what the most control point:
a. Albuminuria
b. HTN
c. Cholesterol

448. In pregnant c/o dysuria, fever which of the following true;


a. Fever is diagnostic.
b. 100,000 colonies collect in aseptic tech almost diagnostic of inf.
c. 10,000-1000 c collected under aseptic technique may diagnostic in asymptomatic
bacteruria
d. Routine screening is indicated for all pregnant women with asymptomatic bacteruria
449. Pregnancy 28 weeks. Complains of fever, flank pain, dysuria next step:
a. Renal US
b. Reassure.
c. Treatment

450. All can cause decreased serum components except:


a. IgA

451. In GN most found:


a. WBC cast.
b. RBC cast

452. Pt after CABG without follow up 0 days came complaining of abdominal pain, big toe
blue, livido reticularis, and creatinine high. Most likely cause:
a. Atheroembolism

453. Patient post catheterization develops high creatinine. Kidney Biopsy showing needle
shaped crystals. The cause:
a. Atheroembolism

454. In minmal change disease, all can be found except:


a. Normal light microscopy
b. Low C3

455. Pt known case of Nephrotic syndrome refused previously kidney Biopsy, transplanted,
after 2year rejection the most common cause of his disease:
a. MNP
b. MCN
c. FSGN
456. In isolated hematuria best treatment:
a. Supportive

457. The main site for Mg+2 reabsorption


a. Thick ascending loop of Henle

458. On treatment with trimethoprim the cause of hyperkalemia:


a. Affect on principle cells on Na channel.

459. Treatment of hyperkalemia:


a. Calcium resonium
b. Salbutamol
c. HD
d. All of the above

460. 42-pt with dehydration Na163, K6.2 best shift K from ECF IS:
a. Insulin + glucose
b. NaHCO3
c. NS.9%

461. Which of the following treatment with NaHCO3 decreases mortality:


a. Alcoholic acidosis
b. Ethylene glycol
c. Heroin

462. Transplantation pt. the diff between Azathioprine & Mycophenolate mofetil
a. Deceases mortality
b. Prolong survival
c. Less episodes of rejection
463. On acute interstitial nephritis difference between NSAID and antibiotic case:
a. Fever and rash 20% in NSAID 80% in antibiotic

464. Young patient with severe hypertension has suspicion of renovascular stenosis:
a. CT scan
b. Doppler US
c. Renal angiogram

465. Best treatment for CKD-4 anemia:


a. Parentral iron
b. Folic acid
c. Recombinant erythropoietin

466. Most common complication of erythropoietin is:


a. Seizure
b. Hypertension

467. Most common stone:


a. Ca oxalate

468. Renal Osteodystrophy comes with:


a. Hypophosphatemia
b. Decrease 1.25 D
c. Secondary hyperparathyroidism

469. Patient with tiredness found creatinine 450 mmol it is associated with:
a. Hypocalcaemia
470. All of the following came with CKD except:
a. Retention of PO4
b. Decrease 1-25 D
c. Increase calcitriol

471. Patient having DM,HTN, still high BP, K 5.5-5.9 best Rx:
a. Propranalol
b. ACEI
c. ARBs
d. Frusemide

472. Waxy cast which true:


a. Should not found in normal person

473. All cause increase erythropoietin except:


a. ADPKD
b. Acquired renal cyst
c. Renal artery stenosis
d. Nephrogenic diabetes insipidus

474. Rate of peritonitis decrease in rate because of;


a. Good selected pt.
b. Good hygiene
c. Treatment a nosocomial carrier of staph aureus
475. Pt with CKD follow up lab Ca10.5, PO4 6.2, PTH 351 best Rx:
a. Sevelamer

476. Patient alcoholic, post head trauma 7 days later develops polyuria Na171, u.osm<160
most common cause:
a. Central diabetes insipidus
b. Nephrgenic diabetes insipidus
c. Cerebral salt wasting

477. Pt operated for hemongioblastoma, later found infiltration Lung, bilateral renal cyst with
mass in the Left kidney BP150/90, pulse 110 beats/min most diagnostic:
a. MRI
b. Renal US
c. 24 hr urine for catecholamines

478. Pt treated for tumor with chemotherapy not treated before with allopurinol, c/o loin pain,
fatigue…lab high creatinine, urea, decrease urine output most next step:
a. IVF
b. PD
c. NaHCO3
d. HD

479. Pt on ICU chest trauma fail chest, develop diarrhea…lab PH 7.39,…the result is:
a. Metabolic acidosis

480. All of the following decease Frusemide response except:


a. Decrease GFR
b. Antibody against Frusemide
c. Increase protein bond to Frusemide on tubule
d. NSAID

481. In Alport's syndrome all true except:


a. AR most inherited type

482. In membranous nephropathy which is most bad prognosis:


a. HTN
b. Male
c. Heavy Proteinuria

483. Pt hypertension receive ACE treatment his creatinine now 115 after 1 wk what next:
a. Reassure inform that reach plateau with 1 month

484. One of the following indicate for urgent dialysis:


a. Slow deterioration of renal function
b. Hyperphosphatemia
c. Sever hyperkalemia

485. In HTN pt the effect of thiazide is:


a. Affect the concentration of distal convoluted tubules

486. Old pt left hemiparesis on thiazide treatment skin turgor, diarrhea. Na135 best treatment:
a. Normal saline

487. A patient with bipolar disorder on lithium treatment develops nephrogenic diabetes
insipidus. The cause will be
a. Lithium blocks ADH receptors
b. Lithium cause acute interstitial nephritis
c. Lithium cause marked diuresis

488. Alport’s syndrome inheritance


a. X-linked

489. All of the following can have small kidneys on USG except
a. Amyloidosis

490. Hypertension is found in how many minimal change disease patients


a. 25%

491. Treatment of choice for DM-1 nephropathy


a. Aspirin
b. Captopril
c. Spironolactone
d. Irbesartan

492. All of the following are nephrotoxic except


a. Amoxicillin
b. Diclofenac
c. Cisplatin
d. Cyclophosphamide

493. All of the following are risk factors for renal stone formation except
a. RTA
b. Gold
c. Cystinosis
d. Dehydration
494. A case of congestive heart failure with hyponatremia. What is the best treatment/cause??
a. Frusemide

495. A case of diabetic nephropathy & hyperkalemia. Best treatment is


a. Salbutamol
b. Calcium gluconate
c. Frusemide
d. Insulin

496. A case of recurrent upper respiratory tract infections, hemoptysis & hematuria. What is
the cause
a. Wegener’s granulomatosis
b. Henoch Schonlein purpura
c. IgA nephropathy
d. Goodpasture's syndrome

497. All of these diseases cause hyperkalemia except


a. Multiple myeloma
498. 26 years old female presents to ER with worsening of fatigue, nausea, and pruritus. She
has HTN. Hb 15, high renal parameters, K=6.5, Na 128. What is the cause
a. ADPKD
b. ………………
c. ……………….

499. What is the best treatment to get rid of hyperkalemia


a. Calcium resonium

500. 9 years old has generalized weakness & diarrhea. Hb 9.3, platelets 28, urea 114,
creatinine 2.3, urine analysis showing hematuria & proteinuria. What is the best treatment
a. IV cyclophosphamide
b. Ciprofloxacin
c. Oral prednisolone
d. Plasma exchange

501. Henoch Schonlein purpura, histopathology


a. Mesangial proliferation

502. Chronic interstitial nephritis


a. Sarcoidosis
b. ……………
c. …………….

503. What is the gold standard for acute interstitial nephritis diagnosis
a. Renal biopsy
b. Urine analysis
c. Serum complement
d. USG abdomen
504. A patient developed rash & impaired RFTs after antibiotics use. What is most likely
diagnosis
a. Acute interstitial nephritis

505. Cyclosporine is associated with all except


a. Chronic interstitial nephritis
b. Hypertension
c. Hirsutism
d. …………..

506. Gentamicin toxicity

507. Analgesic nephropathy can be associated with all except


a. Nephrocalcinosis
b. Malignant hypertension
c. Papillary necrosis
d. Increased risk of transitional cell carcinoma

508. A patient has right sided flank pain radiating to groin & associated urinary symptoms.
Abdominal xray is normal. CT scan showing a 2 cm opacity. What is most likely diagnosis
a. Cystine stone
b. Uric acid stone
c. Calcium oxalate stone

509.
510. PSGN

511. Tuberculosis of urinary tract can present with all except


a. Sterile pyuria
b. Isolation of mycobacteria
c. Ureteral stenosis
d. Painless hematuria
e. Chronic interstitial nephritis

512. An 82 years old man started on HD. What is likely 5 years survival
a. 10-20%
b. 30-40%
c. 50-60%
d. 60-80%
513. A PD patient developed abdominal pain & ascetic fluid is turbid. Analysis revealed
Gram + & Gram – bacteria. He was put on antibiotics. What is the next step
a. Do USG of PD catheter
b. CT scan with contrast
c. MRI
d. Do nothing as diagnosis is almost confirmed

514. A 35 years old male patient with normal BP & no cysts on USG. His grandfather &
uncle have ADPKD. What is the most appropriate statement
a. ADPKD is almost excluded
b. Repeat USG
c. Do MRA brain
d. Do genetic testing
515. A patient DM-1 for 10 years on insulin treatment, developed microalbuminuria & started
on enalapril 6 months ago. Now her RFTs are normal & 24 hours proteinuria is 150 mg. What
will be the effect of pregnancy on renal disease 371 ‫انظر‬
a. No effect
b. Progression to CKD
c. Deterioration of renal function till delivery then improvement after delivery

516. Asymptomatic bacteruria 15 ‫انظر‬


a. Present in 50% of pregnancies
b. Wait until becomes symptomatic
c. Immediate treatment

517. All antibiotics are safe in pregnancy except 31 ‫انظر‬


a. Ampicillin
b. Amoxicillin
c. Cefalexin
d. Nitrofurantoin
e. Quinolones

518. Hypocalcemia is seen in all except 86


a. Hypoparathyoidism
b. Vitamin D deficiency
c. Pseudohypoparathyroidism
d. Grave’s disease
519. A patient has bilateral renal cell carcinoma. Previously, he was operated for brain tumor.
Before tumor resection, what should be done
a. MRI
b. Renal angiography
c. USG
d. Urinary catecholamines

520. Target BP in CKD/ nephropathy is


a. <140/90 mmHg
b. <135/85 mmHg
c. <130/80 mmHg
d. < 125/80 mmHg

521. A patient of SLE with DPGN, taking steroids & cyclophosphamide. Patient is refusing
later due to sterility. What is the alternative 2 ‫انظر‬
a. Azathioprine
b. Mycophenolate
c. Cyclosporine

522. The most common initial presentation of IgA nephropathy is 97


a. Microscopic hematuria
b. Microscopic hematuria with proteinuria
c. Recurrent macroscopic hematuria

523. HSP is associated with all except


a. Abdominal pain
b. Rash
c. Arthralgia
d. Splenomegaly

524. Autosomal recessive, steroids resistant nephrotic syndrome is associated with


a. Nephron
b. Podocin
c. TRCP6
d. Alfa actin 4

525. HIV is associated with


a. Collapsing GN

526. Regarding Minimal change disease all are true except


a. Low C3

527. In nephrotic syndrome, all proteins are decreased except


a. Albumin
b. Ferritin
c. Ceruloplasmin
d. Fibrinogen

528. Decreased complement is not seen in


a. IgA nephropathy

529. Hypocomplemtemia seen in all except


a. Post streptococcal GN
b. Shunt nephritis
c. SLE
d. IgA nephropathy
530. A patient on mechanical ventilator due to flail chest developed diarrhea 8-10 times a
day. pH=7.2, pCO2=30, HCO3. What is the abnormality
a. Metabolic acidosis
b. Respiratory acidosis
c. Metabolic acidosis with metabolic alkalosis
d. Metabolic acidosis with respiratory alkalosis

531. NaHCO3 is helpful in which of the following metabolic acidosis


a. AKI
b. Ethylene glycol poisoning
c. Diabetic ketoacidosis
d. Lactic acidosis

532. A patient taking ifosfamide treatment due to breast cancer. What is side effect
a. Fanconi’s syndrome

533. Side effects of acetazolamide


a. Normal anion gap metabolic acidosis

534. A patient with hypokalemia, metabolic alkalosis with urinary K=10, what is the most
likely cause
a. GI loss

535. Major site of Mg+2 absorption


a. Thick ascending loop of Henle

536. Rhabdomyolysis is associated with all of the following except


a. Hypocalcemia
537. A regular patient on HD came with K= 7.8. What will not be in immediate management
plan
a. Calcium gluconate
b. Insulin glucose infusion
c. NaHCO3
d. Emergent dialysis

538. What will not lower serum K levels


a. Calcium gluconate

539. How trimethoprim causes hyperkalemia


a. By blocking collecting duct apical Na channels

540. ECG findings of hyperkalemia, all are true except


a. Flattening of T wave

541. Patient post CABG, has oliguria, dry skin. Urinary Na 10. What is the most likely
treatment
a. Frusemide
b. Mannitol
c. Fluid expansion
d. Theophyline

542. A 35 years old man denying any H/O nausea, vomiting presented with generalized
weakness. On examination, there is postural drop. BP 100/60 mmHg. Labs K=2.8,
HCO3=33, urinary K= 36. What is the most likely cause
a. Bartter syndrome
b. Gitelman’s syndrome
c. Thiazide abuse
d. Self induced vomiting

543. An elderly stroke patient, hypertensive on thiazides came with obtundation. Mucous
membranes dry low skin turgor. Pulse 120 bpm, BP 80/50 mmHg. What is most appropriate
initial IV fluid
a. 5% DW
b. 5% DW with Normal saline
c. Normal saline
d. Half normal saline

544. A case of reperted upper respiratory tract infection with renal impairment. CXR showing
B/L opacities. Urine analysis showing red cell casts. What is the most likely diagnosis
a. Wegener’s granulomatosis
b. IgA Nephropathy
c. Henoch Schonlein purpura
d. Post streptococcal GN

545. A patient on HD 3/week, has developed ascites. Various investigation…….. what is the
most appropriate treatment
a. Ascitic tap
b. Leeven’s shunt
c. TIPPS
d. Aggressive UF
546. Type-I RTA

547. A young patient started on ACEi. After one week his creatinine 115 mmol/L. what is the
best step
a. Stop ACEi
b. Reassure & continue ACEi
c. Reduce the dose
d. ………….

548. Best way to diagnose CMV in post transplant patients


a. CMV Ab IgG
b. CMV Ab IgM
c. CMV Ag
d. CMV PCR

549. A 62 years man wants to donate his kidney…………………


550. A post transplantation patient developed sudden pain in his big toe. He is taking
azathioprine, steroids & cyclosporine. Uric acid is 8.5. What is the most likely immediate
action
a. NSAIDs
b. Colchicine
c. Allopurinol
d. Decrease dose of cyclosporine

551. Drugs which decreases cyclosporine levels


a. Fluconazole
b. Erythromycin
c. Isoniazide
d. Metocloperamide
e. Nifedipine

552. Mechanism of action of cyclosporine & tacrolimus


a. ………………
b. ……………….
c. ……………….

553. In PD; which statement is most appropriate


a. Icodextrin is more advantageous than glucose
b. Glucose causes UF failure
c. Glucose increases hyperlipidemia
d. Glucose causes obesity
e. Hyperglycemia

554. Renal osteodystrophy includes all except


a. 1,25 dihydroxy Vitamin D3 deficiency
b. Hypocalcemia
c. Hyperphosphatemia
d. 2ndry hyperparathyroidism
e. Decreased Calcitonin

555. Patient having IHD underwent CABG, now deteriorating RFTs & oliguria. What is the
most likely treatment
a. Volume expansion
b. Diuretics
c. Mannitol
d. Hemoperfusion

556. A 72 years old man having oliguria but urea & creatinine are almost normal. What is the
most likely reason
a. Old age
b. Low protein diet
c. Protein energy malnutrition
d. …………………

557. Difference between pre-renal & renal AKI

558. Which one is better indicator of AKI


a. Serum BUN/creatinine ratio
b. Urinary Na
c. Urinary FENa
d. Urine osmolarity

559. Clinical features of atheroembolic renal disease are


a. Livido reticularis
b. Subcutaneous nodules
c. Eosinophilia
d. All of the above

560. A years old diabetic patient on HD for 1 year developed ascites. What is the most
beneficial initial investigation
a. USG abdomen
b. Ascitic tap
c. Viral serology
d. Echocardiography
e. Liver function tests

561. An elderly asymptomatic patient having hypertension & asthma. He is smoker for last
30 years. Came for follow up. Labs showing Na 128. What is the most likely treatment
a. Hypertonic saline
b. Fluid restriction
c. Dextrose water
d. Normal saline

562. Which drug will cause hyponatremia


a. Lithium
b. Chlorpropamide
563. A patient of Nephrotic syndrome, responding well to Frusemide 40mg, but now
increasing dose is required. What is the least likely cause
a. Antibodies to Frusemide

564. Plasma volume is


a. 1/3 of ECF
b. 2/3 of ECF
c. Almost all of ECF
d. Half of ECF

565. A diabetic patient had cardiac arrest during dialysis. What is the most likely cause
a. MI
b. Hypoglycemia
c. Air embolism
d. Excessive UF

566. Patient developed left sided pneumothorax after internal jugular catheter insertion. Chest
tube must be inserted where
a. Right 2nd intercostals space midclavicular line
b. Left 2nd intercostals space midclavicular line
c. Left side ant axillary line
d. Left side posterior axillary line

567. A young patient has hypertension. To confirm the renovascular hypertension, which
investigation you will perform
a. USG kidneys
b. Duplex of renal arteries
c. CT scan of kidneys
d. Renal angiography

568. Child with UTI having features of recurrent reflux nephropathy. What is the
investigation of choice
a. IVU
b. USG
c. CT-Scan
d. Micturating cystourethrogram
e. DMSA scan

569. Best treatment of anemia in CKD-4


a. Oral iron
b. IV iron
c. Folic acid
d. Erythropoietin

570. Granular casts (2 MCQs)

571. Normal sized kidney in all conditions except


a. Amyloidosis
b. Diabetic nephropathy
c. Analgesic nephropathy
d. ADPKD

572. What is the most sensitive test for UTI in children


a. WBCs in urine
b. White cell cast in urine
c. Leukocytosis
d. Fever
e. Leukocyte esterase in urine

573. In case of hypernatremia Na=170 & plasma osmolality 320, the most probable condition
will be
a. Increased intracellular volume increased osmolarity >328
b. Decreased ICV and decreased osmolarity < 328
c. Decreased ICV and increased osmolarity 328
d. No change in ICV and osmolarity 420

574. A 30 weeks pregnant female has BP= 150/100 mmHg. What is the best management
plan
a. Terminate pregnancy
b. Admit to hospital & observe
c. Atenolol
d. …………………..

575. All are side effects of cyclosporine except


a. Hirsutism
b. Interstitial nephritis
c. Tremors
d. Gum hyperplasia
e. …………………….

576. A 62 years old diabetic patient has GFR 17ml/min. He has no contraindication for renal
transplantation. What should be the next management plan
a. PD
b. HD trial
c. Transplantation
d. Wait
577. 12 years old boy dipstick urine +1 protein. What is the next most appropriate
investigation
a. Creatinine clearance
b. 24 hours urinary proteins
c. USG
d. No further investigations

1. 62 years old male with DVT and IVC obstruction due to thrombosis so most like diagnosis
is
a) Nephrotic syndrome
b) SLE
c) Chirstm disease

2. Patient with abdominal pain haematuria, HTN and have abnormality in chromosome 16,
diagnosis is
a) Polycystic kidney

3. Long scenario about patient with polydipsia ad polyuria. Serum osmolarity high.
desmopressin induction no change urine osmolarity and plasma osmolarity so dd is
a) Nephrognic type DI
b) central type DI

nephrogenic DI.

reatment:
4. Female presented with thirst and polyuria, all medical history is negative and she is not
known to have medical issues, she gave history of being diagnosed as Bipolar and on Lithium
but her Cr and BUN is normal. What is the cause of her presentation?
a) Adverse effect of lithium
b) Nephrogenic DI
c) Central DI

5. Adenosine dose should be reduced in which of the following cases :


a) Chronic renal failure.
b) Patients on thiophyline.

6. Adult polycystic kidney disease is inherited as:


a) Autosomal dominant
b) Autosomal recessive
c) X linked

7. IVP study done for a male & showed a filling defect in the renal pelvis non-radio opaque.
U/S shows echogenic structure & hyperacustic shadow. The most likely diagnosis is:
a) Blood clot
b) Tumor
c) Uric acid stone

8. Patient came with HTN, KUB shows small left kidney, arteriography shows renal artery
stenosis, what is the next investigation?
a) Renal biopsy
b) Renal CT scan
c) Renal barium
d) Retrograde pyelography

9. Best way to diagnose post streptococcus Glomerulonephritis (spot diagnosis):


a) Low C3
b) RBC casts

10. Female patient did urine analysis shows epithelial cells in urine, it comes from:
a) Vulva
b) Cervix
c) Urethra
d) Ureter

11. Female with history of left flank pain radiating to groin, symptoms of UTI, what is
diagnosis?
a) Appendicitis
b) Diverticulitis
c) Renal colic

12. Pre-Renal Failure:


a) Casts
b) Urine Osm < 400
c) Urine Na < 20 mmol/L
d) Decreased water excretion
e) Hematuria

Urine osm<400 in intrinsic renal failure but >500 in pre-renal failure


-renal failure if >200 it is intrinsic renal failure
ure

13. Patient with history of severe hypertension, normal creatinine, 4g protein 24 hrs. Right
kidney 16cm & left kidney 7cm with suggesting of left renal artery stenosis. Next
investigation:
a) Bilateral renal angiography
b) Right percutaneous biopsy
c) Left percutaneous biopsy
d) Right open surgical biopsy
e) Bilateral renal vein determination

14. All of them are renal complications of NSAIDs except:


a) Acute renal failure
b) Tubular acidosis
c) Interstitial nephritis
d) Upper GI bleeding
NSAIDs but upper GI bleeding is not renal complication!

15. Patient has bilateral abdominal masses with hematuria, what is the most likely diagnosis?
a) Hypernephroma
b) Polycystic kidney disease

16. Acute Glomerulonephritis, all are acceptable Investigations except:


a) Complement
b) Urinanalysis
c) ANA
d) Blood culture
e) Cystoscopy

17. 20 years old female present with fever, loin pain & dysuria, management include all of
the following except:
a) Urinanalysis and urine culture
b) Blood culture
c) IVU (IVP)
d) Cotrimexazole
-hydration.

18. Urine analysis will show all EXCEPT:


a) Handling phosphate.
b) Specific gravity.
c) Concentrating capacity.
d) Protein in urine.

19. In acute renal failure, all is true EXCEPT:


a) Phosphatemia.
b) Uremia.
c) Acid phosphate increases.
d) K+ increases.

20. Old patient, bedridden with bacteremia “organism is enterococcus fecalis”, what the
source of infection?
a) UTI
b) GIT

21. A 56 years old his CBC showed, Hb=11, MCV= 93, Reticulocyte= 0.25% the cause is:
a) Chronic renal failure
b) Liver disease
c) Sickle cell anemia
d) G6P dehydrogenase deficiency

22. 30 years old with repeated UTIs, which of the following is a way to prevent her
condition?
a) Drink a lot of fluid
b) Do daily exercise

23. 65 years old presented with acute hematuria with passage of clots and left loin and scrotal
pain. the Dx
a) Prostatitis
b) Cystitis
c) Testicular cancer
d) Renal cancer
24. Patient has saddle nose deformity, complaining of SOB, haemoptysis and haematuria.
most likely diagnosis is:
a) Wagner’s granulomatosis

25. Old patient complain of urinary incontinence. Occur at morning and at night without
feeling of urgency or desire of micturition, without exposure to any stress, what is the
diagnosis?
a) Urgency incontinence
b) Urge incontinence
c) Stress incontinence
d) Over Flow incontinence
26. Heavy smoker came to you asking about other cancer, not Lung cancer, that smoking
increase its risk:
a) Colon
b) Bladder
c) Liver

27. The most common cause of secondary hypertension is:


a) Renal artery stenosis
b) Adrenal hyperplasia
c) Pheochromocytoma
d) Cushing's disease

28. The most common cause of chronic renal failure:


a) HTN
b) DM
c) Hypertensive renal disease
d) Parenchymal renal disease
e) Acute glomerulonephritis

29. Male patient present with prostatitis (prostatitis was not mentioned in the question),
culture showed gram negative rodes. The drug of choice is:
a) Ciprofloxacin “Floqinolon”
b) Ceftriaxone
c) Erythromycin
d) Trimethoprime
e) Gentamicin
30. Patient complaining of left flank pain radiating to the groin, dysuria and no fever. The
diagnosis is:
a) Pyelonephritis
b) Cystitis
c) Renal calculi

31. A 3 weeks old baby boy presented with a scrotal mass that was transparent & non-
reducible. The diagnosis is:
a) Hydrocele
b) Inguinal hernia
c) Epidydimitis

32. Uncomplicated UTI treatment:


a) TMP-SMX for 3 days
b) Ciprofloxacin 5 days

33. A 29 years old man complaining of dysuria. He was diagnosed as a case of acute
prostatitis. Microscopic examination showed gram negative rods which grow on agar yeast.
The organism is:
a) Chlamydia.
b) Legionella
c) Mycoplasma

34. Patient with renal transplant, he developed rejection one week post transplantation, what
could be the initial presentation of rejection?
a) Hypercoagulability
b) Increase urine out put
c) Fever
d) Anemia

35. Patient with hematuria and diagnosed with bladder cancer. What’s the likely causative
agent?
a) Schistosoma haematobium

36. Most common manifestation of renal cell carcinoma is:


a) Haematuria
b) Palpable mass
c) HTN

37. Patient with excessive water drinking and frequent urinate, FBS 6.8 diagnosis up to now:
a) Normal blood sugar
b) IFG
c) DM 2
d) D. insepidus

38. Patient with DKA the pH=7.2, HCO3=5, K=3.4 the treatment:
a) Insulin 10 U
b) 2 L NS
c) 2 L NS with insulin infusion 0.1 U/kg/hr

39. 6 years old presented with cola colored urine with nephritic symptoms the First test you
would like to do:
a) Renal function test
b) Urine microscopic sedimentation
c) Renal ultrasound

40. Young adult presented with painless penile ulcer rolled edges, what next to do?
a) CBC
b) Darkfeild microscopy
c) Culturing

41. Diabetic female her 24h-urine protein is 150mg


a) start on ACEIs
b) refer to nephrologist
c) Do nothing , this is normal range

42. Patient with flank pain, fever ,vomiting, treatment is


a) Hospitalization and intravenous antibiotics and fluid
nt hospitalization

43. Elderly patient complaining of urination during night and describe when he feel the
bladder is full and need to wake up to urinate, he suddenly urinate on the bed this is:
a) Urgency incontinence
b) Urge incontinence
c) Stress incontinence
d) Flow incontinence

44. The best test for renal stones:


a) CT without contrast
45. 70 years old male patient with mild urinary dripping and hesitancy, your diahnosis is mild
BPH. What is your next step in management?
a) transurethral retrograde prostatectomy
b) Start on medication
c) open prostatectomy

46. Patient with dysuria, frequency and urgency but no flank pain, what is the treatment?
a) Ciprofloxacin po once daily for 3-5 days
b) Norfocin PO OD for 7 – 14 days

47. UTI >14 day, most probably cause pyelonephritis


a) 0,05%
b) 0,5%
c) 5%
d) 50%

48. Man have long history of urethral stricture present with tender right testis & WBC in
urine so diagnosis is
a) Epididymo-orchitis
b) Testicular torsion
c) varicocele

49. None opaque renal pelvis filling defect seen with IVP, US revels dense echoes & acoustic
shadowing,what is the most likely diagnosis?
a) Blood clot
b) Tumor
c) Sloughed renal papilla
d) Uric acid stone
e) Crossing vessels
magnesium-ammonium-
phosphate

50. an opaque renal pelvis filling defect seen with IVP, US revels dense echoes & acoustic
shadowing , The MOST likely diagnosis:
a) Blood clot
b) Tumor
c) Sloughed renal papilla
d) Uric acid stone
e) Crossing vessels

51. Complication of the rapid correction of hypernatremia:


a) Brain edema

52. Epididymitis one is true :


a) The peak age between 12-18 years
b) U/S is diagnostic
c) Scrotal content within normal size
d) Typical iliac fossa pain
e) None of the above
-40 year

affected side as well as increased blood flow or scrotalabscess

53. The most important diagnostic test for Previous Q is :


a) Microscopic RBC
b) Macroscopic RBC
c) RBC cast.

54. 17 year old male presented to you with history of abdominal pain and cramps in his leg he
vomited twice, his past medical history was unremarkable. On examination he looks
dehydrated with dry mucous membranes, His investigation: Na: 155 mmol/l, K: 5.6 mmol/l ,
Glucose; 23.4 mmol/l, HCO3: 13, Best tool to diagnose this condition is:
a) Plain X-ray
b) Ultrasound
c) Gastroscopy
d) Urine analysis (Dipstick analysis)

55. Patient come abdominal pain and tender abdomen with hypernatremia and hyperkalemia
and vomiting and diarrhea, what is the next investigation:
a) Urine analysis

56. Patient present with URTI, after 1 week the patient present to have hematuria and edema,
what is most probably diagnosis?
a) IgA nephropathy
b) Post streptococcus GN

57. Most common cause of ESRD:


a) HTN
b) DM
-stage renal disease includes :
1) Kidney disease – obviously ESRD starts as early kidney disease .
2) Diabetic nephropathy - 43.2% of kidney failure is due to diabetes
3) Chronic kidney failure -ESRD is by definition the last state of chronic kidney failure
4) Hypertension - 23% of cases
5) Glomerulonephritis - 12.3% of cases
6) Polycystic kidney disease - 2.9% of cases

58. BPH all true except :


a) Prostatitis
b) Noctouria
c) Hematuria
d) Urine retention
e) Diminished size &strength of stream

ptied after finishing

59. The most accurate to diagnose acute Glomerulonephritis is:


a) RBC cast in urine analysis
b) WBC cast in urine analysis
c) Creatinine level increase
d) Shrunken kidney in US
e) Low Hgb but normal indices

60. 75 years old man came to ER complaining of acute urinary retention. What will be your
initial management:
a) Send patient immediately to OR for prostatectomy
b) Empty urinary bladder by Folley’s catheter and tell him to come back to the clinic
c) Give him antibiotics because retention could be from some sort of infection
d) Insert Foley’s catheter and tell him to come to clinic later
e) Admission, investigations which include cystoscopy

61. Regarding group A strept pharyngitis what is true


a) Early treatment decrease incidence of post strept GN

62. The investigation of high sensitivity and specificity of urolithiasis :


a) IVP
b) X-RAY abdomen after CT scan
c) US
d) MRI
e) Nuclear scan.

63. Patient with PID there is lower abdominal tenderness, on pelvic exam there is small mass
in..… Ligament,what is the treatment?
a) Colpotomy
b) Laparotomy
c) laparoscopy

64. Young male patient with dysuria fever and leukocytosis, PR indicate soft boggy tender
prostate, Dx :
a) Acute prostatitis
b) Chronic Prostatitis
c) Prostatic CA

65. 60 years old male known to have (BPH) digital rectal examination shows soft prostate
with multiple nodularity & no hard masses, the patient request for (PSA) for screening for
prostatic cancer what will you do?
a) Sit with the patient to discuss the cons &pros in PSA test
b) Do trans-rectal US because it is better than PSA in detection
c) Do multiple biopsies for different sites to detect prostatic ca

66. The most likely cause of gross hematuria in a 35 years old man is :
a) Cystitis
b) Ureteral calculi
c) Renal carcinoma
d) Prostatic carcinoma
e) Bladder carcinoma

67. Concerning urinary calculi, which one of the following is true?


a) 50% are radiopaque
b) 75% are calcium oxalate stones
c) An etiologic factor can be defined in 80% of cases
d) A 4-mm stone will pass 50% of the time
e) Staghorrn calculi are usually symptomatic

stones.

68. Benign prostatic hypertrophy:


a) TRUSS is better than PSA
b) No role in PSA
c) PSA role
d) Biopsy
69. An 80 year old male presented with dull aching loin pain & interrupted voiding of urine.
BUN and creatinine were increased. US revealed a bilateral hydronephrosis. What is the most
probable diagnosis?
a) Stricture of the urethra
b) Urinary bladder tumor
c) BPH
d) Pelvic CA
e) Renal stone

70. The best investigation for kidney function :


a) 24 h collect urine
b) Creatinine clearance

71. Patient have DM and renal impairment when he had diabetic nephropathy: there is curve
for albumin
a) 5 years
b) 10 years
c) 20 years
d) 25 years
by 5-10 years. Once proteinuria is
detected, renal function gradually deteriorates over 10-15 years

72. Benign prostatic hyperplasia , all are true EXCEPT:


a) Parotitis
b) Nocturia
c) diminished size and strength of stream
d) hematuria
e) urine retention

73. In Testicular torsion, all of the following are true, except


a) Very tender and progressive swelling.
b) More common in young males.
c) There is hematuria
d) Treatment is surgical.
e) Has to be restored within 12 hours or the testis will infarct.

74. 50 years old patient complaining of episodes of erectile dysfunction, history of stress
attacks and he is now in stress what you will do?
a) Follow relaxation strategy
b) Viagra
c) Ask for investigation include testosterone

75. Premature-ejaculation, all true except:


a) Most common sexual disorder in males
b) uncommon in young men
c) Benefits from sexual therapy involving both partners
d) It benefit from anxiety Rx
☻ Premature ejaculation (PE) is the most common sexual dysfunction in men younger than
40 years.

76. Refer for surgical prostatectomy Best laboratory finding in urinalysis for active
glomerulonephritis is :
a) High creatinine
b) RBC cast
c) WBC cast
77. Pyelonephritis :
a) Discharge on oral antibiotic
b) Admission + fluid + antibiotic

78. 10 years old boy woke up at night with lower abdominal pain, important area to check:
a) kidney
b) lumbar
c) rectum
d) Testis

79. Patient present with testicular pain, O/E: bag of worms, what is the diagnosis?
a) Varicocele

80. Old man presented with tender and enlarged prostate and full bladder. Investigations
show hydronephrosis. What is the likely diagnosis?
a) Acute Renal Failure
b) Bladder Cancer
c) BPH

81. A patient with gross hematuria after blunt abdominal trauma has a normal-appearing
cystogram after the intravesical instillation of 400 ml of contrast. You should next order:
a) A retrograde urethrogram.
b) An intravenous pyelogram.
c) A cystogram obtained after filling, until a detrusor response occurs.
d) A voiding cystourethrogram.
e) A plain film of the abdomen after the bladder is drained.
82. Patient will do cystoscope suffer from left hypocondrial pain
a) Refer to vascular surgery
b) Refer to urologist

83. Old patient complaining of hematuria, on investigation, patient has bladder calculi, most
common causative organism is:
a) Schistosoma
b) CMV

84. Old man with urinary incontinence, palpable bladder after voiding, urgency & sense of
incomplete voiding dx;
a) Stress incontinence
b) Overflow incontinence
c) Reflex incontinence
d) Urge incontinence

85. Young male with 3 day of dysuria, anal pain , O/E per rectum boggy mass :
a) Acute prostatitis

86. Radiosensitive testicular cancer:


a) Yolk sac
b) Seminoma
c) Choriocarcinoma.

87. DM HTN patient with MI receiving metformin and diltiazem and other medication his
creatine clearance is high, you will do:
a) add ACE inhibitor
b) remove metformin (contraindicated in renal failure)
c) continue same medication

88. Old patient with diarrhea dehydration corrected with 3 liters of D5, later he became
confused with headache. Most probable cause:
a) Hyponatremia
b) Hypernatremia
c) Hypokalemia
d) Hyperkalemia

89. The best test for renal stones:


a) CT without contrast

90. Old lady healthy with signs of UTI she took a lot of medication but no improvement on
exam mild bladder tenderness everything else is normal?
a) Diabetic nephropathy
b) Interstitial nephropathy

91. Old lady with signs of UTI medication didn`t work with her after exam there is mass?
a) Colpoctomy

92. Patient is pale, diaphoretic has left flank pain and vomiting, his abnormal labs are low K
2.3, high CL 114, low HCO3 15, urea n, Na n, urine PH 6.5
a) metabolic acid
b) metabolic alkalosis
c) respiratory alkalosis
d) respiratory acidosis
Renal tubular acidosis (RTA) is composed of a group of disorders characterized by an
inability of the kidney to resorb bicarbonate/secrete hydrogen ions, resulting in
hyperchloremic, normal anion gap metabolic acidosis. Renal function (glomerular filtration
rate [GFR]) must be normal or near normal

93. All the following cause hyponatremia except:


a) DKA
b) Diabetes insipidus “causes Hypernatremia due to huge loss of water in the form of diluted
urine”
c) High vasopressin level
d) Heart failure

94. Patient with HTN presented with edema, azotemia, GFR: 44 (not sure about - 5) what is
the cause of her Kidney disease?
a) bilateral renal artery stenosis
b) diabetic nephropathy
c) Reflux..
d) Renal tubular acidosis

95. Uric acid in body how the body removed by


a) increase metabolism of uric acid in liver
b) increase execration of uric acid in urine
c) execration of uric acid by lung

96. All the following are side effect of thiazide diuretics except:
a) Has diabetogenic effect
b) Cause hypocalemia
c) cause hypomagnesimia
d) Flat curve response
e) cause Hypokalemia
f) It causes Hypercalcemia

97. When lactic acid accumulates, body will respond by:


a) Decrease production of bicarbonate
b) Excrete CO2 from the lungs
c) Excrete Chloride from the kidneys
d) Metabolize lactic acid in the liver

extent by
hyperventilation, via medullary chemoreceptor, leading to ↑ removal of CO2 in the lung

98. Old age man, feel that the voiding is not complete and extreme of urine not strong and by
examination there is moderate BPH and PSA = 1ng/ ml what you will do?
a) Surgery

99. Old patient has loin pain with high serum urea and creatinine, U/S showed bilateral
hydronephrosis, what is the most common cause?
a) Ureteral stricture
b) Retroperitoneal fibrosis
c) Bladder neoplasm
d) Prostate cancer
100. Patient with hypertension, renography shows right kidney 14 cm left kidney 7 cm..
Arteriogram shows renal artery stenosis in left. What to do next??
a) arteriogram
b) renogram
c) CT scan
d) Biopsy

101. What is the best investigation regarding renal function:


a) Serum creatinine level
b) Insulin Level

102. Patient has frequent urgency to urinate at night time and when she wakes up to go to WC
she found herself wet. What does she have?
a) Urge incontinence
b) stress incontinence
c) Overflow incontinence.

103. Female with dysuria, urgency and small amount of urine passed .she received several
courses of AB over the last months but no improvement, all investigations done urine
analysis and culture with CBC are normal, you should consider:
a) Interstitial cystitis
b) DM
c) Cervical erosion
d) Candida albicans

104. US showed enlarged both kidneys with multiple sized spaces & patient has trisomy of
chromosome 16
a) Polycystic kidney disease
105. Old patient with dehydration corrected with 3 liters of D5, later he became confused
with headache. Most probable cause:
a) Hyponatremia
b) Hypernatremia
c) Hypokalemia
d) Hyperkalemia

S-ar putea să vă placă și